CPA Exam - REG

अब Quizwiz के साथ अपने होमवर्क और परीक्षाओं को एस करें!

The built-in gains tax applicable to S corporations was enacted to prevent a tax avoidance plan applicable to: individual shareholders. C corporations. another S corporation. all flow-through entities.

C corporations. Built-in gains tax was enacted to prevent C corporations planning on selling or distributing property from electing S corporation status before the sale or distribution. Electing S status would otherwise avoid the double taxation associated with C corporations. The built-in-gains tax imposes income tax on any built-in-gain accrued before an S election becomes effective, for a period of years following the effective date of the election.

Which of the following is allowed in the calculation of the taxable income of a simple trust? Exemption Standard deduction Brokerage commission for purchase of tax-exempt bonds Charitable contribution

Exemption Estates and trusts are separate taxable entities. A personal exemption of $300 is allowed for a trust that is required to distribute all of its income currently (simple trust). A simple trust has no beneficiaries that are charitable organizations.

Green is self-employed as a human resources consultant and reports on the cash basis for income tax purposes. Select the appropriate tax treatment on Form 1040 (U.S. Individual Income Tax Return) for the share of ordinary income from an investment in a limited partnership reported in Form 1065, Schedule K-1. Taxable as other income on Form 1040 Reported in Schedule B, Interest and Dividend Income Reported in Schedule C as trade or business income Reported in Schedule E, Supplemental Income and Loss

Reported in Schedule E, Supplemental Income and Loss A partner's share of ordinary income from an investment in a limited partnership reported in Form 1065, Schedule K-1 should be reported in Schedule E, Supplemental Income and Loss.

Which of the following statements related to a traditional IRA is not correct? The maximum deduction is generally $6,000 per individual. An individual at least age 50 may deduct up to $7,000. A 10% penalty tax applies to any withdrawal made prior to age 59-1/2. The $6,000 maximum deduction is phased out for taxpayers with adjusted gross income over certain amounts.

A 10% penalty tax applies to any withdrawal made prior to age 59-1/2. The 10% penalty does not apply to all withdrawals made prior to age 59-1/2. Withdrawals may be made for qualified education expenses and by first-time homebuyers with no penalty.

Which of the following is a component of the general business credit? Work opportunity tax credit Earned income tax credit Foreign income tax credit American Opportunity Tax Credit

Work opportunity tax credit The earned income tax credit, foreign income tax credit, and American Opportunity Tax Credit are all types of tax credits, but they are not included in the general business tax credit category. The work opportunity tax credit is one of more than two dozen credits accounted for as part of the general business tax credit.

A 33-year-old taxpayer withdrew $30,000 (pretax) from a traditional IRA. The taxpayer has a 33% effective tax rate and a 35% marginal tax rate. What is the total tax liability associated with the withdrawal? $10,000 $10,500 $13,000 $13,500

$13,500 When the 33-year-old taxpayer withdraws $30,000 from an IRA, a penalty of 10% is imposed (0.10 × $30,000 = $3,000). The $30,000 is added to taxable income and will be taxed not at the effective rate but at the marginal rate of 35% (0.35 × $30,000 = $10,500). The total tax liability as a result of the withdrawal is $13,500 ($3,000 + $10,500).

Owners of at least 10% of a residential rental unit who actively participate in the rental can deduct losses up to $25,000 a year. However, a phaseout of the deduction begins at $100,000 of adjusted gross income. The deduction of the rental loss is completely phased out at what level? $125,000 $150,000 $175,000 $200,000

$150,000 For every $2 of adjusted gross income (AGI) over the $100,000 level, one dollar of that real estate loss is phased out, so that at $150,000 in AGI, the full deduction is phased out.

A partner in a real estate partnership had a basis of $5,000 at the beginning of the year and a basis of $10,000 at year-end. The partner's at-risk amount at year-end was $8,000. The partner's Schedule K-1 listed $12,000 as the partner's share of the partnership's ordinary loss. What amount can the partner deduct on the partner's tax return? $5,000 $8,000 $10,000 $12,000

$8,000 Losses from the active conduct of a trade or business are limited to amounts invested by the taxpayer for which the taxpayer is "at risk." There may be other limitations, but no loss is allowable unless the taxpayer invested assets or is unconditionally obligated to pay debts of the trade or business activity. In this question, the partner's at-risk amount is $8,000 and the partner is limited to that deduction.

In the current year, George Giant paid the following taxes: $500 in automobile registration $5,000 in property taxes $3,800 in New Mexico income tax How much of these taxes could potentially be deducted on George's Schedule A? $3,800 $5,500 $9,300 $8,800

$9,300 Beginning in 2018, taxpayers are limited to deducting $10,000 of state taxes—property and state and local income tax. Since the total paid is less than $10,000, they are all deductible for the current year ($500 + $5,000 + $3,800 = $9,300).

Partnership - Treatment of an increase in liabilities of the partnership

-Treated as though the partner contributed money for a share of those liabilities -Basis of the partner's investment increases

Start-up expenses - Special Rules

-$5,000 deduction permitted as a current business expense* *reduced by the amount the taxpayer's total start-up costs exceed $50,000; remaining costs must be amortized ratably over a 180-month period as a Section 197 intangible

Banks Corp., a calendar-year corporation, reimburses employees for properly substantiated qualifying business meal expenses. The employees are present at the meals, which are neither lavish nor extravagant, and the reimbursement is not treated as wages subject to withholdings. What percentage of the meal expense may Banks Corp. deduct in 2021? 0% 80% 100% 50%

100% For tax years beginning after 1993, only 50% of business meals are deductible provided there is a documented business purpose. The Consolidated Appropriations Act (CAA) allows businesses a 100% deduction for business-related meals and beverages provided by a restaurant. This provision is available for 2021 and 2022 tax years.

What percentage of a self-employed person's medical insurance premiums is deductible "above the line" to arrive at adjusted gross income (AGI)? None 60% 70% 100%

100% If you're self-employed and your business has a net profit, you can deduct 100% of your medical, health, and qualified long-term care insurance premiums for yourself, your spouse, and your dependents. The insurance can also cover your child who was under the age of 27 at the end of the year, even if the child was not your dependent.

In the current year, Louise, a self-employed consultant, finances a new vehicle that is used 75% for business and 25% for personal use. How much, if any, of the interest incurred on the loan could be deductible in arriving at adjusted gross income? 0% 25% 75% 100%

75% Louise could treat 75% of the interest as deductible in arriving at her net business income and would treat the remaining 25% as nondeductible personal interest.

A trust in which the beneficiaries are given a future right to trust income or corpus and the $15,000 gift tax exclusion is retained is termed a: reversionary trust. Crummey trust. gift-leaseback. throwback trust.

Crummey trust. A Crummey trust is a "safe harbor" rule that allows the annual gift tax exclusion on gifts to a trust. The gift of a future interest provision does not apply to gifts to a Crummey trust if trust assets go to the beneficiaries on or before age 21. Giving the right to the trust income (generally only up to $15,000 per donee, per year) meets the requirement even though the beneficiaries do not actually withdraw the funds.

Green is self-employed as a human resources consultant and reports on the cash basis for income tax purposes. Select the appropriate tax treatment on IRS Form 1040 (U.S. Individual Income Tax Return) for interest expense on a home-equity line of credit for an amount borrowed to finance Green's business. Not deductible as a business expense 50% deductible on Form 1040 to arrive at adjusted gross income Fully deductible on Schedule C—Profit or Loss From Business Partially deductible on Schedule C—Profit or Loss From Business

Fully deductible on Schedule C—Profit or Loss From Business Interest expense can be deducted from business income on Schedule C if the interest is used for the trade or business of the taxpayer, subject to limitations. This business deduction is limited to the sum of business interest income, 30% of adjusted taxable income, and floor plan financing interest; however, this limitation does not apply to small business taxpayers (gross receipts less than $26 million in the past three consecutive years), electing real property trades, and electing farming businesses. Since the home equity loan is used entirely for business purposes, the full deduction for interest expense should be allowed, assuming Green qualifies for the small business taxpayer exception.

Which of the following circumstances may permit the piercing of the corporate veil of a closely held corporation and thus may cause its shareholders to be held personally liable? I. The corporation is thinly capitalized. II. The corporation borrows money from a shareholder without giving the shareholder a security interest in corporate assets. I only II only Both I and II Neither I nor II

I only A corporation is a separate, legal entity that is separate from its shareholders, directors, officers, and employees. Thus, owners have liability for the organization limited to their investment in the organization. Piercing the corporate veil means that a shareholder, director, or officer can be held personally liable for corporate obligations. In order for the "lifting" of the corporate veil to occur, two elements must be present: 1. A shareholder, director, or officer must have controlled the corporation for his or her own benefit in an attempt to protect himself or herself from legal liability. 2. A shareholder, director, or officer must have used the corporation in an improper manner, doing such things as perpetuating fraud, not capitalizing the organization adequately, or looting the corporation of assets.

Under the Uniform Partnership Act, which of the following statements is (are) correct regarding the effect of the assignment of an interest in a general partnership? I. The assignee is personally responsible for the assigning partner's share of past and future partnership debts. II. The assignee is entitled to the assigning partner's interest in partnership profits and surplus on dissolution of the partnership. I only II only Both I and II Neither I nor II

II only Assignment transfers rights under a contract to another (not liabilities). The assignee obtains the right to the assignor's share of partnership profits and what the assignor would receive if the partnership would be dissolved.

Jones is a household employer who owes both FICA and FUTA tax in relation to the employment of his domestic service employees (who are nonagricultural employees). In addition, he has withheld income tax and FICA tax from the domestic employees' wages. Which of the following statements best describes the forms that Jones may file in order to report all applicable federal employment taxes and withheld taxes for his domestic employees? Form 940 (Employer's Annual Federal Unemployment (FUTA) Tax Return) Form 941 (Employer's Quarterly Federal Tax Return) Form 1040, Schedule H (Household Employment Taxes) Jones must file both Form 940 and 941 even if he files Schedule H. Jones must file Form 941 to report the withheld tax even if he files Schedule H; he does not have to file Form 940 if he files Schedule H. Jones must file Form 940 to report the FUTA tax even if he files Schedule H; he does not have to file Form 941 if he files Schedule H. If Jones files Schedule H, he does not have to file Form 940 or Form 941.

If Jones files Schedule H, he does not have to file Form 940 or Form 941. Form 1040, Schedule H may be used to report federal employment taxes on cash wages paid to household employees. Federal employment taxes that may be paid with Schedule H include Social Security, Medicare, withheld federal income, and federal unemployment (FUTA) taxes. Thus, if Jones files Schedule H, he does not need to file Form 940 or Form 941. IRS Instructions, Schedule H (Form 1040); IRS Notice 95-18

Which of the following statements is correct if a taxpayer agrees to changes made during an examination and he signs an agreement, but does not pay the taxes due? If the taxpayer does not pay the additional tax when he or she signs the agreement, the taxpayer will receive a bill that includes interest and an additional penalty. If the taxpayer pays the amount due within 21 calendar days of the billing date and the amount is less than $100,000, the taxpayer will not have to pay more interest or penalties. If the taxpayer pays when he or she signs the agreement, the interest is generally waived from the due date of the return to the date of the payment. None of the answer choices are correct.

If the taxpayer pays the amount due within 21 calendar days of the billing date and the amount is less than $100,000, the taxpayer will not have to pay more interest or penalties.

Partnership - Partnership basis of contributed of personal assets (nonbusiness property)

Lesser of 1) adjusted basis to the contributing partner 2) FMV at the time the property was contributed to the property

Partnership - Distribution - Cash < Basis of partnership interest

No gain is recognized

On March 23, Year 6, Tom Lewis sold 50 shares of ABC Corp. stock at a $3,500 loss. He had purchased the stock three years earlier. He repurchased 50 shares of ABC Corp. on April 15, Year 6. Tom had no other stock transactions for the year. Select the appropriate tax treatment for the capital loss. Not deductible $3,500 long-term capital loss deductible $3,500 short-term capital loss deductible $3,500 long-term capital loss, limited to $3,000 deductible

Not deductible A loss sustained upon a sale or other disposition of stock or securities is not allowed if, within a period beginning 30 days before the date of the sale or disposition and ending 30 days after that date, the taxpayer has acquired, or has entered into a contract or option to acquire, substantially identical stock or securities. This is known as the "wash sale rules."

Tom Lewis, an individual taxpayer and employee of ABC Corp., paid insurance premiums this year that covered the possible loss of earnings resulting from disability. Select the appropriate tax treatment for the payment of these premiums. Not deductible on Form 1040 Deductible in full on Schedule A—Itemized Deductions Deductible on Schedule A—Itemized Deductions, subject to a threshold amount of 2% of adjusted gross income Deductible on Schedule A—Itemized Deductions, subject to a limitation of 50% of adjusted gross income

Not deductible on Form 1040 Premiums paid for insurance against an individual's possible loss of earnings are not deductible under the Internal Revenue Code.

Which of the following taxes are not deductible as an itemized deduction by a self-employed taxpayer? Real estate taxes Personal property taxes Foreign income taxes One-half of self-employment taxes

One-half of self-employment taxes One-half of self-employment taxes are an adjustment to adjusted gross income and are not included on Schedule A as an itemized deduction.

For head of household filing status, which of the following costs are considered in determining whether the taxpayer has contributed more than one-half the cost of maintaining the household? A Food consumed in the home B Value of services rendered in the home by the taxpayer Both A and B Neither A nor B Only A Only B

Only A In determining the cost for maintaining a household, only paid costs should be considered. Those costs can include mortgage or rent expense, utility charges, food consumed in the home, and repair and maintenance costs. Any unpaid costs (such as the value of services) are not considered when determining the cost of maintaining the home.

Wages paid for domestic services are subject to special rules for determining whether they are subject to payroll taxes. For 2021, at what level do domestic wages become subject to Social Security and Medicare tax? Over $2,300 to one employee in a year Over $2,200 total wages in a year Over $1,000 total wages in a quarter Only if requested by employee

Over $2,300 to one employee in a year

Partnership - Current year net income (loss)

Partner's distributive share of partnership income and gain (including tax-exempt income) - Partner's distributive share of partnership loss and deductions (including nondeductible, non-capital expenditures) = Current year net income (loss)

A seven-person partnership lacks a partnership agreement. Under the Revised Uniform Partnership Act, how many votes are required to approve an extraordinary transaction of partnership business? Four votes Five votes Six votes Seven votes

Seven votes Seven votes is correct. The Revised Uniform Partnership Act (RUPA) contains a list of five extraordinary acts that require unanimous consent of the partners. In other words, a majority vote is not sufficient. An extraordinary transaction is an example of a matter requiring unanimous agreement of all the partners. Therefore, in a seven-person partnership, all seven partners would have to approve the transaction. The remaining answer choices (four, five, and six votes) are incorrect because these vote thresholds fall shy of constituting unanimous consent as required under RUPA.

A spouse died on December 31, year 1. The couple had no dependents. What should be the filing status of the surviving spouse in year 2? Single Married filing jointly Qualifying widow(er) Head of household

Single In order to qualify as qualifying widow(er), the individual must have a child or stepchild whom they are able to claim as a dependent. Since there were no dependents in this question, the filing status of the surviving spouse is single.

How is the depreciation deduction of nonresidential real property, placed in service in this year, determined for regular tax purposes using MACRS? Straight-line method over 40 years 150% declining-balance method with a switch to the straight-line method over 27.5 years 150% declining-balance method with a switch to the straight-line method over 39 years Straight-line method over 39 years

Straight-line method over 39 years Nonresidential real property is any land or building which is rented or leased to a tenant for other than residential purposes; it includes office buildings, stores, and warehouses. For regular tax, nonresidential real property placed in service on or after May 13, 1993, is depreciated over 39 years using the straight-line method of depreciation and applying the half-month convention. IRC Section 168(c)

Partnership - Other increases/(decreases)

Sum of all other increases and decreases that affect the partner's capital account

Under the UCC Secured Transactions Article, which of the following actions will best perfect a security interest in a negotiable instrument against any other party? Filing a security agreement Taking possession of the instrument Perfecting by attachment Obtaining a duly executed financing statement

Taking possession of the instrument Under the Uniform Commercial Code (UCC) Secured Transactions Article, perfection of a security interest in a negotiable instrument is achieved only by the creditor taking possession of the instrument.

Mike and Jane Lewis, a married couple, file a joint federal income tax return. They have one child, age 15, whom they support 100%. Both are under age 65. They have the following income and expenses for the year: Mike's wages $65,000 Jane's wages 60,000 Total allowable itemized deductions 13,000 Mike's contribution to an IRA 4,000 Jane's contribution to an IRA 4,000 Mike is not covered by a pension plan at work, while Jane is covered by a plan at her employer. Assume that the standard deduction amount for married filing jointly is $24,000, and the IRA contribution limit is $120,000. What is the Lewises' taxable income amount? $96,000 $96,400 $100,000 $101,000

The Lewises' taxable income amount is calculated as follows: Mike's wages $ 65,000 +Jane's wages 60,000 =Adjusted gross income $125,000 Less: Standard deduction (24,000) Taxable income $101,000 Taxpayers can elect each year to itemize deductions or take the standard deduction. If itemized deductions are $13,000, the Lewises will take the standard deduction of $24,000 for married, filing jointly. Mike may not take an IRA deduction because they are over the threshold in AGI. Jane's IRA contribution is not deductible since they are above the threshold of $120,000 in AGI and she is covered by a pension plan at work.

What is the foundational document of the bankruptcy law? The federal bankruptcy code Various state laws that were made uniform in the bankruptcy code The United States Constitution The common law

The United States Constitution Bankruptcy results when the intended performance of the debtor becomes impossible due to excessive debt. The solution is to take the debtor's property and distribute it to the unpaid creditors through a uniform process. The United States Constitution has a specific provision allowing Congress to enact bankruptcy law.

Which of the following circumstances best describes a landlord's constructive eviction of a tenant who has a written lease for the property? The landlord starts a legal proceeding against the tenant for failure to pay rent. The landlord sues the tenant because the tenant complained to a government agency about the condition of the premises. The landlord refuses to provide utilities to the tenant. None of the answer choices are examples of constructive eviction.

The landlord refuses to provide utilities to the tenant. In the law of real property, constructive eviction describes a situation in which a landlord either does something or fails to do something that the landlord has a legal duty to provide, rendering the property uninhabitable. Refusing to provide utilities to the tenant is an example of a constructive eviction. To maintain an action for damages, the tenant must show that the uninhabitable conditions were a result of the landlord's actions (not the actions of a third party) and that the tenant vacated the premises in a reasonable time. The tenant can claim the same legal remedies available to a tenant who was actually told by the landlord to leave.

Under the Tax Cuts and Jobs Act of 2017 (TCJA), a new qualified business income (QBI) deduction was created, commonly known as a Section 199A deduction. How is QBI defined? The net amount of income and gains from the operations of a business, including investment income The net amount of income and gain from the operations of a business, excluding investment income The net amount of income, gain, deductions, and loss from the operation of a business, excluding investment income The net amount of income, gain, deductions, and loss from the operation of a business, including investment income

The net amount of income, gain, deductions, and loss from the operation of a business, excluding investment income Between 2018 and 2025, individuals will get a deduction of 20% of qualified business income (QBI) from pass-through entities, such as partnerships, limited liability companies, S corporations, and sole proprietorships (including a sole-member limited liability company). QBI is the net amount of income, gain, deduction, and loss from the operation of a business, not including investment income.

Which of the following statements is correct concerning the voluntary filing of a petition in bankruptcy? If the debtor has 12 or more creditors, the unsecured claims must total at least $16,750. The debtor must be insolvent. If the debtor has less than 12 creditors, the unsecured claims must total at least $16,750. The petition may be filed jointly by spouses.

The petition may be filed jointly by spouses. A voluntary filing of bankruptcy may be done jointly by a husband and wife if both consent. The debtor does not have to be insolvent. There is no requirement as to the number of creditors or the amount of unsecured claims.

Which of the following statements is correct regarding a shareholder's right to inspect corporate books and records? The right is absolute. The right is conditioned upon the demanding shareholder owning at least $5,000 worth of stock. The right requires that the demand to inspect be for a proper purpose. The right exists only when fraud or illegality is alleged.

The right requires that the demand to inspect be for a proper purpose. Under the Model Business Corporation Act, a shareholder's right to inspect corporate books and records is not absolute. A shareholder may inspect corporate books and records when the inspection is for a proper purpose. In addition, the right of inspection may be limited to shareholders who own at least 5% of the corporation's stock or have owned their stock for at least six months. The proper purpose need not include fraud or illegality. Contrast this question with the absolute rights of a shareholder, which include the right to view the articles of incorporation, bylaws, shareholder lists, and minutes of the shareholder meetings from the past three years. Note: State law differs widely in this area because not all states have adopted the Model Business Corporation Act and some states have regulations more advanced than the Act.

To properly create an inter vivos trust funded with cash, the grantor must: execute a written trust instrument. transfer the cash to the trustee. provide for payment of fees to the trustee. designate an alternate trust beneficiary.

The written trust instrument would be the starting place for creating any trust. The written trust instrument would provide for payment of fees to the trustee and designate any and all beneficiaries. Transferring the cash to the trustee would complete the creation of the trust.

"Hot assets" of a partnership would include which of the following? Cash Unrealized receivables Section 1231 assets Capital assets

Unrealized receivables The "hot assets" of a partnership include the unrealized receivables and inventory. These are the items that would generate ordinary income. Both Section 1231 assets and capital assets generate capital gains. Cash creates neither ordinary income nor capital gains.

In the absence of an election to adopt an annual accounting period, the required tax year for a partnership is: a tax year that results in the greatest aggregate deferral of income. a calendar year. a tax year of one or more partners with a more than 50% interest in profits and capital. a tax year of a principal partner having a 10% or greater interest.

a tax year of one or more partners with a more than 50% interest in profits and capital. Generally, a partnership must adopt a tax year that is used by a partner or partners who hold more than a 50% interest in the partnership. If this rule does not apply, then the partnership must use the tax year of all of its principal partners (holding 5% or more interest in partnership capital or profits). If that rule does not apply, the partnership must use the tax year that results in the least (not greatest) aggregate deferral of income. Generally, it is an S corporation (not a partnership) that must use a calendar year.

A distinction between a surety and a co-surety is that only a co-surety is entitled to: reimbursement (indemnification). subrogation. contribution. exoneration.

contribution. A distinction between a surety and co-surety is that only a co-surety is entitled to "contribution." Contribution occurs when the other co-sureties are obliged to contribute their share of the liability to the surety that had to make the full payment. Generally, each surety agrees to a maximum liability, and any amount that is paid that is less than the full debt results in each surety being liable for the percentage of their agreed liability to the total debt. Reviewing the other terms: Reimbursement is the right of the co-sureties to be paid by the principal debtor if they have to make good on their guarantee to the creditor. Both sureties and co-sureties have this right. Subrogation gives the sureties and co-sureties the right to any assignments of property in the event that they have had to make payment to the creditor. Exoneration gives both sureties and co-sureties the right to compel the principal debtor to make payment to the creditor if it possibly can.

Able hired Carr to restore Able's antique car for $800. The terms of their oral agreement provided that Carr was to complete the work within 18 months. Actually, the work could be completed within one year. The agreement is: unenforceable, because it covers services with a value in excess of $500. unenforceable, because it covers a time period in excess of one year. enforceable, because personal service contracts are exempt from the statute of frauds. enforceable, because the work could be completed within one year.

enforceable, because the work could be completed within one year. If it is possible to complete a contract within one year, the contract is enforceable. The statute of frauds requires a contract to be in writing if the contracts which cannot be completed within one year. Since this contract can be completed within one year, the statute of frauds does not require it to be in writing and the oral contract is enforceable. If the contract could not be performed within a year, it would be necessary for the contract to be written and signed for it to be enforceable.

Carr Corp. declared a 7% stock dividend on its common stock. The dividend: must be registered with the SEC pursuant to the Securities Act of 1933. is includable in the gross income of the recipient taxpayers in the year of receipt. has no effect on Carr's earnings and profits for federal income tax purposes. requires a vote of Carr's stockholders.

has no effect on Carr's earnings and profits for federal income tax purposes. A "stock dividend" is the issuance by the company of additional shares of stock to current shareholders. The issuance has no effect on the company's earnings and profits for income tax purposes.

A $50,000 cash gift will reduce an individual's unified credit against the gift tax if the gift is made: to a university located in the United States. in trust for a 14-year-old child living in the United States. to a spouse who is a U.S. citizen. to a church located in the United States.

in trust for a 14-year-old child living in the United States. The unified credit is a credit that allows donors and decedents to transfer a limited amount of property without being subject to the gift or estate tax. (For 2021, the exemption amount for the gift tax is $11,700,000.) This represents the amount of property that a taxpayer may transfer free of the gift tax. Gift tax paid on taxable gifts given during the taxpayer's lifetime reduces the applicable credit against the estate tax. The IRS defines a gift as cash or property given to another individual or business. A donation differs from a gift in that it is a cash or property transfer to a qualified charitable organization, such as a church. Gifts to a spouse are not taxable and are unlimited. As long as the gift to a university is for tuition, it is also not taxed.

Maco, Inc., and Kent contract for Kent to provide Maco certain consulting services at an hourly rate of $20. Kent's normal hourly rate was $90 per hour, the fair market value of the services. Kent agreed to the $20 rate because Kent was having serious financial problems. At the time the agreement was negotiated, Maco was aware of Kent's financial condition and refused to pay more than $20 per hour for Kent's services. Kent has now sued to rescind the contract with Maco, claiming duress by Maco during the negotiations. Under the circumstances, Kent will: win, because Maco refused to pay the fair market value of Kent's services. win, because Maco was aware of Kent's serious financial problems. lose, because Maco's actions did not constitute duress. lose, because Maco cannot prove that Kent, at the time, had no other offers to provide consulting services.

lose, because Maco's actions did not constitute duress. Contracts can be voided due to invalid consent if there was duress. Duress must be compelled by fear. There is no evidence of fear, only of financial difficulties.

On June 5, 20X1, Gold rented equipment under a 4-year lease. On March 8, 20X2, Gold was petitioned involuntarily into bankruptcy under the Federal Bankruptcy Code's liquidation provisions. A trustee was appointed. The fair market value of the equipment exceeds the balance of the lease payments due. The trustee: may not reject the equipment lease because the fair market value of the equipment exceeds the balance of the lease payments due. may elect not to assume the equipment lease. must assume the equipment lease because its term exceeds one year. must assume and subsequently assign the equipment lease.

may elect not to assume the equipment lease. Under the U.S. Bankruptcy Code, a trustee in a Chapter 7 liquidation may elect not to assume the equipment lease even though the fair market value of the equipment exceeds the balance of the lease payments due. Recall that a liquidation bankruptcy means that all assets will be sold and proceeds made available to pay creditors. Continuing to pay on a lease of equipment for a business that is being sold off would not meet the trustee's obligation to maximize distributions (payments) to general (unsecured) creditors.

Under the liquidation provisions of Chapter 7 of the Federal Bankruptcy Code, certain property acquired by the debtor after the filing of the petition becomes part of the bankruptcy estate. An example of such property is: municipal bond interest received by the debtor within 180 days after the filing of the petition. alimony received by the debtor within one year after the filing of the petition. Social Security payments received by the debtor within 180 days after the filing of the petition. gifts received by the debtor within one year after the filing of the petition.

municipal bond interest received by the debtor within 180 days after the filing of the petition. Upon filing a Chapter 7 bankruptcy proceeding, certain property becomes part of the estate if it is received (or entitled to be received) within 180 days of the filing. This type of property includes gifts, inheritances, various forms of property settlements (including divorce property divisions but not alimony) and life insurance proceeds. Notice the answers included most all of the included items, but had a different time schedule for their receipt. Notice that the time line alone is not controlling. Via statute, various exemptions exist (up to certain dollar amounts), including such items as limited interests in homeowners equity, motor vehicles, household goods, tools of the trade, health aids, personal injury claims, alimony, and certain pension benefits. Finally, the bankruptcy statute specifically excludes Social Security payments.

A partner's interest in specific partnership property is: assignable to the partner's individual creditors. subject to attachment by the partner's individual creditors. both assignable to and subject to attachment by the partner's individual creditors. neither assignable to nor subject to attachment by the partner's individual creditors.

neither assignable to nor subject to attachment by the partner's individual creditors. A partner's interest in specific partnership property is not assignable to the partner's individual creditors. A partner's interest in specific partnership property is not subject to attachment by the partner's individual creditors. A creditor may not obtain a partner's interest in specific partnership property of the partnership. A court will not award a partner's creditor with a lien on specific partnership property.

Debt collectors are prohibited from doing all of the following except: provide written verification of the debt. communicate with a third party by postcard. disclose in correspondence that the sender is in the debt collection business. identify the debt collector's employer unless expressly requested.

provide written verification of the debt. When contacting third parties, debt collectors are prohibited from doing all of the following: disclosing that the debtor owes a debt; communicating with the third party (or debtor) by postcard; disclosing in correspondence that the sender is in the debt collection business; or identifying the debt collector's employer unless expressly requested. Additionally, collectors may not harass, oppress, or abuse the debtor; and may not falsely misrepresent that they are affiliated with the government, are attorneys, have sold the debtor's account to another, or will take actions that they cannot lawfully take.

The term "active participation" for a passive activity loss is relevant in relation to: rental real estate activities. working interests in oil and gas properties. passive activities in which the taxpayer materially participates. passive activities in which the taxpayer does not materially participate.

rental real estate activities. In general, passive activity losses in excess of passive activity income are not allowed. There is an exception to the rule for rental real estate activities; if an individual actively participated in a rental real estate activity, they may be able to deduct up to $25,000 of passive activity loss from nonpassive income. Participation in management decisions such as new tenant approval, rental terms, repairs, and capital expenditures is sufficient to meet the "active participation" definition. The passive activity rules apply to individuals, estates, trusts, personal service corporations, and closely held C corporations. The passive activity loss rules do not apply to partnerships, widely held C corporations, or S corporations.

Fern purchased property from Nix for $150,000. Fern obtained a $90,000 loan from Jet Bank to finance the purchase, executing a promissory note and mortgage. By recording the mortgage, Jet protects its: priority against a previously filed real estate tax lien on the property. priority against all parties having earlier claims to the property. rights against the claims of subsequent bona fide purchasers for value. rights against Fern under the promissory note.

rights against the claims of subsequent bona fide purchasers for value. By recording the mortgage, Jet protects its rights against the claims of subsequent bona fide purchasers for value. The whole purpose of recording mortgages is to let the world know—or give notice—of a claim against the property by the lender. Filing a mortgage does not give a party rights against lienholders who have previously filed their claims against the property. Jet would not receive a priority against a previously filed real estate tax lien on the property, nor would Jet receive a priority against all parties having earlier claims to the property.

Annualization is required for a short tax period to ensure: all income is reported. the appropriate annual marginal tax rate applies. deductions are not overstated. shifting income from one period to another does not occur.

the appropriate annual marginal tax rate applies. A short tax period is required to prevent taxpayers from receiving the benefit of having lower taxable income as the result of a tax year shorter than 12 months. By annualizing, the taxpayer must use the marginal rates applicable to what income would have been for 12 months.

Downs, Frey, and Vick formed the DFV general partnership to act as manufacturers' representatives. The partners agreed Downs would receive 40% of any partnership profits and Frey and Vick would each receive 30% of such profits. It was also agreed that the partnership would not terminate for five years. After the fourth year, the partners agreed to terminate the partnership. At that time, the partners' capital accounts were as follows: Downs, $20,000; Frey, $15,000; and Vick, $10,000. There also were undistributed losses of $30,000. If Frey died before the partnership terminated: Downs and Vick, as a majority of the partners, would have been able to continue the partnership. the partnership would have continued until the 5-year term expired. the partnership would automatically dissolve. Downs and Vick would have Frey's interest in the partnership.

the partnership would automatically dissolve. If Frey died before the partnership terminated, the partnership would automatically dissolve. Under the typical state partnership laws, death automatically dissolves a partnership. If the partners wanted to continue the partnership after the death of a partner, they can agree to form a new partnership.

A husband and wife can file a joint return even if: the spouses have different tax years, provided that both spouses are alive at the end of the year. the spouses have different accounting methods. either spouse was a nonresident alien at any time during the tax year, provided that at least one spouse makes the proper election. they were divorced before the end of the tax year.

the spouses have different accounting methods. No joint return shall be made if the husband and wife have different taxable years, except in cases where the taxable year ends on different days because of the death of either or both. A nonresident alien individual married to a citizen of the United States may file a joint return only if both make an election to file jointly. The determination of whether an individual is married shall be made as of the close of the taxable year. There is no exception for disallowance of filing jointly as a result of spouses using different accounting methods.

Dunne and Cook signed a contract requiring Cook to rebind 500 of Dunne's books at 80¢ per book. Later, Dunne requested in good faith, that the price be reduced to 70¢ per book. Cook agreed orally to reduce the price to 70¢. Under the circumstances, the oral agreement is: enforceable, but proof of it is inadmissible into evidence. enforceable, and proof of it is admissible into evidence. unenforceable, because Dunne failed to give consideration, but proof of it is otherwise admissible into evidence. unenforceable, due to the statute of frauds, and proof of it is inadmissible into evidence.

unenforceable, because Dunne failed to give consideration, but proof of it is otherwise admissible into evidence. Generally, the parol evidence rule will not allow oral evidence to alter the terms of a written contract. The parol evidence rule will only allow the admission of evidence that is not in the written contract in limited circumstances.

Tom Lewis, a single taxpayer, received $1,000 in gross receipts for renting his lake cabin for 10 days during the year. The expenses related to this rental included: Newspaper ad for rental $100 Cleaning and maintenance of rental 200 Tom's total income on his individual tax return will be increased by what amount as a result of the rental activities? $0 $1,000 $900 $700

$0 If an individual rents a personal residence for a period of time less than 15 days during a tax year, the rents he or she receives are not included in gross income and the associated expenses are not deductible as a rental expense. Thus, since Tom rented the property for only 10 days all year, he will not show any income or expenses from the rental on his tax return.

Rona Corp.'s alternative minimum taxable income was $200,000. What tax is owed? $12,500 $0 $27,500 $52,500

$0 The Tax Cuts and Jobs Act of 2017 (TCJA) repealed the alternative minimum tax (AMT) for corporations.

In Year 9, Smith paid $6,000 to the tax collector of Wek City for realty taxes on a two-family house owned by Smith's mother. Of this amount, $2,800 covered back taxes for Year 8, and $3,200 covered Year 9 taxes. Smith resides on the second floor of the house, and his mother resides on the first floor. In Smith's itemized deductions on his Year 9 return, what amount was Smith entitled to claim for realty taxes? $6,000 $3,200 $3,000 $0

$0 The legal owner of the property is Smith's mother. Smith is not entitled to a deduction to realty taxes as he is not legally obligated to pay. This applies even though Smith resides in the property.

Wood's residence had an adjusted basis of $150,000 and it was destroyed by a tornado. An appraiser valued the decline in market value at $175,000. Later that same year, Wood received $130,000 from his insurance company for the property loss and did not elect to deduct the casualty loss in an earlier year. Wood's adjusted gross income was $60,000 and he did not have any casualty gains. What total amount can Wood deduct as an itemized deduction for the casualty loss, after the application of the threshold limitations? $38,900 $19,900 $0 $13,900

$0 Under the Tax Cuts and Jobs Act of 2017 (TCJA), for tax years 2018 through 2025, the personal casualty and theft loss deduction is not available, except for casualty losses incurred in a federally declared disaster. The Taxpayer Certainty and Disaster Relief Act of 2019 removes the requirement that taxpayers itemize to get any disaster loss deduction but does nothing to expand qualified disaster losses beyond those from presidentially declared disasters.

Baker, a sole proprietor CPA, has several clients that do business in Spain. While on a 4-week vacation in Spain, Baker took a 5-day seminar on Spanish business practices that cost $700. Baker's round-trip airfare to Spain was $600. While in Spain, Baker spent an average of $100 per day on accommodations, local travel, and other incidental expenses, for total expenses of $2,800. What amount of educational expense can Baker deduct on Form 1040, Schedule C, Profit or Loss From Business? $700 $1,200 $1,800 $4,100

$1,200 For an international trip, airfare would not be deductible if the trip is primarily personal, as indicated by the word "vacation." (IRC Section 274) The cost of Baker's course of $700 would be deductible and the expenses of $100 per day for five days would also be deductible for an additional $500 and a total of $1,200. Note: When international travel is both personal and business, but not primarily personal, the deductible airfare is pro-rated by deducting (the number of days used for business divided by the total number of days traveled) times the round-trip airfare.

Stone owns 100% of an S corporation and materially participates in its operations. Her stock basis at the beginning of the year is $5,000. During the year, the corporation makes a distribution of $3,500 and passes through a loss from operations of $2,000 for the year. What loss can Stone deduct on her personal tax return? $0 $1,500 $2,000 $5,500

$1,500 A taxpayer's loss deduction from an S corporation is limited to amounts "at risk" in a trade or business or income-producing activity under IRC Section 465 losses. The amount at risk is the total of the taxpayer's basis in the S corporation plus any loans that the taxpayer has made to the S corporation. In this case, the basis begins as $5,000. The "ordering rules" require basis to be reduced first by the $3,500 distribution, leaving a remaining basis of $1,500. Therefore, the allowed loss is $1,500.

Mary From, single, owned rental real estate which generated a tax loss of $60,000. Mary materially participated in the rental activity. Mary's adjusted gross income before considering the $60,000 loss was $130,000. What amount of the loss can offset income from nonpassive sources? $15,000 $10,000 $0 $60,000

$10,000 Rental activities are considered passive activities even if the taxpayer materially participates. However, for rental real estate activities, a loss up to $25,000 may be deducted. However, the $25,000 loss must be reduced by half of the adjusted gross income (before the loss) in excess of $100,000. Thus, the deduction is $10,000 ($25,000 − (0.50 × $30,000)).

An individual is a 50% partner who materially participates in Stone Partnership. The individual's adjusted basis at the beginning of the year was $0. Stone had a $70,000 loss from its business. Stone borrowed $30,000 from a bank, of which $20,000 remained unpaid at year-end. What amount of loss is the individual allowed in the current year from Stone? $35,000 $15,000 $10,000 $0

$10,000 The individual is allowed a current-year loss of $10,000 from Stone. Increases in the liabilities of the partnership are treated as though the partner contributed money for a share of those liabilities. The basis of the partner's investment increases accordingly. A partner's basis is decreased by the distributive share of partnership losses (including capital losses); a partner's basis may not be decreased below zero. A 50% partner would increase the basis of his account by $10,000 (50% × $20,000) from the amount of the loan that is still outstanding, and would decrease his basis by his half of the $70,000 loss provided he has sufficient basis. Since his basis is only $10,000, he is only allowed a $10,000 loss in the current year.

An individual taxpayer reports the following information: U.S. Treasury bond income $ 100 Municipal bond income 200 Rental income 500 Investment interest expense 1,000 What amount of investment interest can the taxpayer deduct in the current year? $100 $300 $800 $1,000

$100 In general, in the case where a taxpayer borrows money to buy property that is held for investment, the interest that the taxpayer pays is investment interest and it can be deducted up to the amount of investment income. However, a person cannot deduct interest incurred to produce tax-exempt income (municipal bonds) and passive activity or rental real estate activity (rental income). Investment income generally includes gross income from property held for investment such as interest, dividends, annuities, and royalties. In this case, the maximum deductible interest expense is limited to the $100 of income included in gross income from the U.S. Treasury bonds.

Carroll is 35 years old and an unmarried taxpayer with an adjusted gross income of $100,000. Carroll incurred and paid the following unreimbursed medical expenses: Doctor bills resulting from a serious fall: $5,000 Cosmetic surgery that was necessary to correct a congenital deformity: $15,000 Carroll had no medical insurance. For regular income tax purposes, what was Carroll's maximum allowable medical expense deduction, after the applicable threshold limitation that is in effect for 2021? $0 $12,500 $7,500 $10,000

$12,500 For 2021, Carroll is only allowed to deduct the amount of medical expenses that exceed 7.5% of Carroll's adjusted gross income (AGI). Carroll's AGI is $100,000; therefore, Carroll has to exclude the first $7,500 ($100,000 × 0.75). Carroll should only deduct $12,500 ($15,000 + $5,000 - $7,500) on the return. Note that cosmetic surgery is deductible only when it is necessary to correct a congenital abnormality (as in Carroll's case), personal injury resulting from an accident or trauma, or disfiguring disease. (IRC Section 213(d)(9)) Note: Under the Tax Cuts and Jobs Act of 2017 (TCJA), medical and dental care expenses in excess of 10% of AGI are deductible if the taxpayer itemizes. The SECURE Act of 2019 changed the medical deduction floor percentage from 10% to 7.5% for 2020, and the Consolidated Appropriations Act, 2021 (CAA) made the 7.5%-of-AGI floor permanent.

A taxpayer reported the following in a tax year: Salary $122,000 Capital gain dividends 3,700 Partnership short-term capital loss (6,300) The taxpayer acquired the partnership interest during the year in exchange for a capital contribution of $2,750, and there were no additional items affecting the taxpayer's basis in the partnership. What is the taxpayer's adjusted gross income for the year? $119,400 $122,000 $122,700 $122,950

$122,950 The taxpayer's adjusted gross income is $122,950. Since the taxpayer's basis in the partnership is limited to his contribution of $2,750, the taxpayer is not entitled to deduct the full short-term capital loss on his return. He may only take into consideration the first $2,750 of short-term capital loss with the remaining $3,550 being carried over until the taxpayer acquires more basis in the partnership. On his individual tax return, the short-term capital gain recognized would be $950 ($3,700 − $2,750). Therefore, this amount is added to the taxpayer's salary, resulting in an adjusted gross income of $122,950 ($122,000 + $950).

JJ, a small-scale farmer, had the following receipts in the current year: Cash received from farm stand sales $5,600 Cash received from selling crops to mill 4,000 Cash received from selling produce to a restaurant 5,000 Funds received for pledged crop sale in the next year 1,000 How much should JJ include in his taxable gross income for the current year? $9,600 $14,600 $0 $15,600

$14,600 A cash-basis farmer includes in gross income all cash receipts from the sale of crops and produce raised along with the sale of livestock. The income from a sale of a crop should be included in the year the crop is actually sold ($5,600 + $4,000 + $5,000 = $14,600). In this case, the pledged crops ($1,000) are not yet sold and should be included in the following tax year.

Smith (single filing status) has an adjusted gross income (AGI) of $120,000 without taking into consideration $40,000 of losses from rental real estate activities. Smith actively participates in the rental real estate activities. What amount of the rental losses may Smith deduct in determining taxable income? $0 $15,000 $20,000 $40,000

$15,000 Individuals (and married taxpayers filing jointly) may offset up to $25,000 of ordinary income with losses from rental real estate activities. This exemption is reduced (but not below zero) by 50% of the amount by which the adjusted gross income (AGI) of the taxpayer for the year exceeds $100,000. (Note that married taxpayers filing separately may each offset $12,500, reduced (but not below zero) by 50% of the amount by which the AGI of each taxpayer exceeds $50,000.) Therefore, $25,000 − (($120,000 − $100,000) × 0.50) = $15,000 deduction allowed.

Aztec, a C corporation, distributed an asset to Burn, a shareholder. The asset had a fair market value of $30,000 and was subject to a $40,000 liability, assumed by Burn. The asset had an adjusted basis of $25,000. What amount of gain must Aztec recognize? $0 $5,000 $10,000 $15,000

$15,000 The general rule is that when a corporation distributes an appreciated asset to a shareholder, the corporation is treated as though it had sold the asset to the shareholder for the asset's fair market value. However, if the asset is subject to a liability that exceeds the asset's fair market value, the corporation is treated as though it sold the asset for the amount of the liability. Aztec is treated as if it sold the asset for $40,000 and recognizes gain on the difference between that and its basis of $25,000. Aztec recognizes gain of $15,000, computed as follows: Deemed sales price based on liability in excess of FMV $40,000 -Tax basis 25,000 =Gain recognized $15,000 IRC Sections 311(b) and 336(b)

A taxpayer employed full-time as an engineer has the following income items: Self-employment income $50,000 Rental income 15,000 Dividend income 2,000 Long-term capital gain 1,500 Short-term capital loss 1,000 What amount is the taxpayer's passive income? $2,500 $15,000 $17,500 $18,500

$15,000 The taxpayer's passive income is $15,000. Passive income activities include the following: A trade or business in which the taxpayer does not materially participate Rental activities (with some exceptions) Any activity in which the taxpayer's liability is limited Therefore, of the income items listed, only rental income ($15,000) would qualify as passive income. The remaining answer choices ($2,500, $17,500, and $18,500) are incorrect because they include one or more sources of income that would be classified as nonpassive income items.

Rich is a cash-basis, self-employed air-conditioning repairman with gross business receipts of $20,000. Rich's cash disbursements were as follows: Air-conditioning parts $2,500 Yellow Pages listing 2,000 Estimated federal income taxes on self-employment income 1,000 Business long-distance telephone calls 400 Charitable contributions 200 What amount should Rich report as net self-employment income? $15,100 $14,900 $14,100 $13,900

$15,100 Rich should report $15,100 as self-employment income. His gross self-employment income of $20,000 can be reduced by eligible business deductions for air-conditioning parts of $2,500, the Yellow Pages listing of $2,000, and business long-distance calls of $400, which results in a net self-employment income of $15,100. Estimated federal income tax on self-employment income cannot be considered as a deduction in determining net self-employment income. Charitable contributions are an itemized deduction on Schedule A of IRS Form 1040 or page 1 of Form 1040 for nonitemizers and for years 2020 and 2021. Charitable contributions are not considered in determining self-employment income.

Aqua Corp. had an operating income of $500,000 and operating expenses of $350,000 in the current year. Aqua had a long-term capital gain of $30,000 and a $50,000 short-term capital loss. What is Aqua's taxable income for the current year? $130,000 $147,000 $150,000 $180,000

$150,000 Aqua has a net short-term capital loss of $20,000 ($50,000 - $30,000). Capital losses can only be used to offset capital gains and can then be carried back three years and carried forward for five years. Aqua's taxable income is $150,000 ($500,000 - $350,000); none of the $30,000 capital gain is included as it is fully offset by the capital loss.

On December 1, Year 5, Krest, a self-employed cash-basis taxpayer, borrowed $200,000 to use in her business. The loan was to be repaid on November 30, Year 6. Krest paid the entire interest amount of $24,000 on December 1, Year 5. What amount of interest was deductible on Krest's Year 5 income tax return? $0 $2,000 $22,000 $24,000

$2,000 This question is testing the prepaid interest rules for cash-basis taxpayers. A cash-basis taxpayer is generally not allowed to deduct payments for interest that will accrue in future periods. Only the amount of interest that is properly accrued in the current year is allowed. In this case, there was $24,000 interest paid for a 12-month loan. The current-year deduction would be computed as follows: $24,000 ÷ 12 months = $2,000 per month The loan was outstanding for one month of the tax year, so $2,000 is deductible. The balance would be carried forward and could be deducted next year.

Moore, a single taxpayer, had $50,000 in adjusted gross income for Year 5. During Year 5, she contributed $18,000 to her church. She had a $10,000 charitable contribution carryover from her Year 4 church contribution. What was the maximum amount of properly substantiated charitable contributions that Moore could claim as an itemized deduction for Year 5? $10,000 $18,000 $25,000 $28,000

$28,000 The maximum cash contribution a taxpayer may deduct in each tax year is limited to 60% of the taxpayer's adjusted gross income (AGI) so long as the recipient charities are qualified for this ceiling: 60% × $50,000 AGI = $30,000 $10,000 carryover from Year 4 + $18,000 for Year 5 = $28,000 (with $0 carryover to Year 6) For 2020 and 2021, the limitation for cash contributions is 100% of AGI. In this case, the taxpayer is entitled to a full deduction under both sets of rules (2020-2021 and other years).

John Evert exchanged land held as an investment for other land to be held as an investment. Relevant data is: Property Given by John Basis $60,000 Value 90,000 Mortgage on land 10,000 Property Received by John Value $65,000 Cash 15,000 What is John's recognized gain or loss on the exchange? $15,000 $25,000 $30,000 $55,000

$30,000 Value received $65,000 +Mortgage relief 10,000 +Cash received 15,000 Amount realized $90,000 Less: Basis given 60,000 Realized gain $30,000 The realized gain is recognized to the extent of the boot (mortgage relief and cash received) of $25,000. In a like-kind exchange, the basis of property received is the basis of the property given up plus any gain recognized, plus boot (cash or property not of a like kind, including mortgage relief) paid, less any loss recognized, less boot received. (Note that under the Tax Cuts and Jobs Act (TCJA), like-kind exchanges are applicable only to business and investment real estate; like-kind exchanges of personal property are no longer eligible for deferment of gains.)

Roland and Wanda Czewick have one child who is 14 years old. The Czewicks are divorced and, under the divorce agreement, Wanda claims the child as her dependent in the current year. Wanda's adjusted gross income (AGI) for the current year is $70,000. Wanda files as head of household. What is the child tax credit amount Wanda can claim in the current year? $350 $600 $750 $3,000

$3,000 The Tax Cuts and Jobs Act of 2017 (TCJA) raised the child tax credit (CTC) to $2,000, with a phaseout beginning at $200,000. The American Rescue Plan Act of 2021 (ARPA) increases the CTC to $3,000 per child or $3,600 for children under age 6 as of the end of the tax year. A phaseout applies to the temporarily increased amounts for 2021, meaning the $1,600 amount per child under age 6 ($3,600 increased CTC amount in 2021 less $2,000 "normal" CTC amount) or the $1,000 amount per child age 6 or older ($3,000 increased CTC amount in 2021 less $2,000 "normal" CTC amount). The increased CTC is phased out at a rate of $50 for each $1,000 of modified AGI over the threshold. For single individuals, this threshold is $75,000. So, under ARPA, Wanda may qualify for an increased credit of $3,000.

Alex, single, incorporated his business by investing $80,000 for which he received Section 1244 stock. Alex also loaned the corporation $20,000. The corporation became bankrupt and Alex lost the entire $100,000. What should Alex report as a result of the bankruptcy? $80,000 capital loss, $20,000 ordinary loss $50,000 capital loss, $50,000 ordinary loss $100,000 ordinary loss $100,000 capital loss

$50,000 capital loss, $50,000 ordinary loss A loss from Section 1244 stock is treated as an ordinary loss (up to $50,000 if single). Any loss in excess of $50,000 is treated as a capital loss.

Garner is a 25% partner in Classic General Partnership. On February 3, Garner's tax basis in Classic was $10,000 when she received a nonliquidating distribution of $5,000 cash. Classic had no unrealized receivables, appreciated inventory, or properties that had been contributed by its partners. Classic reported the following for the same year: U.S. Treasury interest $ 30,000 Ordinary business income 120,000 What amount of income from Classic should Garner include in her gross income for that year? $7,500 $30,000 $37,500 $42,500

$37,500 Nonliquidating distributions of a partnership are taxed as a capital gain to the partner only on the excess over the partner's basis and result in a reduction in the partner's capital account. Partnerships are not taxable entities—they function as a conduit for income tax purposes. Ordinary income and losses along with special gain and loss items flow through the partnership down to the individual partners, who report these items on their personal tax returns. Garner should report $37,500 as her gross income [($30,000 + $120,000) × 25%]. (Note: Interest on U.S. Treasury securities is fully taxable at the federal level.)

The partnership of R and S had an ordinary loss during the current year of $10,000. R and S are general partners who actively and materially participate in the daily operations of the partnership. R and S share profits and losses equally. R had an adjusted basis of $4,000 for his interest before taking into account the current-year loss. On his personal tax return R may deduct: $5,000 ordinary loss. $5,000 capital loss. $4,000 ordinary loss and $1,000 capital loss. $4,000 ordinary loss.

$4,000 ordinary loss. A partner's share of a partnership loss is limited to the partner's basis in the partnership. It is then limited by passive activity rules and at-risk rules under IRC Sections 469 and 465. The excess of the loss over the basis is carried forward.

Mane Bank lent Eller $120,000 and received securities valued at $30,000 as collateral. At Mane's request, Salem and Rey agreed to act as uncompensated co-sureties on the loan. The agreement provided that Salem's and Rey's maximum liability would be $120,000 each. Mane released Rey without Salem's consent. Eller later defaulted when the collateral held by Mane was worthless and the loan balance was $90,000. Salem's maximum liability is: $30,000. $45,000. $60,000. $90,000.

$45,000. The correct answer is $45,000. This problem tests your understanding of the legal concept of co-sureties. Co-sureties are two or more parties who agree to be liable for the debt of the principal. Unless a contract stipulates otherwise, co-sureties share in the liability. In this problem, Salem and Roy agreed to be co-sureties for Eller. When Eller defaulted and the co-sureties became liable for $90,000, each surety would be liable for half the debt of $45,000. Since the creditor released one surety without the consent of the other, the remaining surety is not liable for any portion of the released surety's share.

Tom Lewis, a single taxpayer, had the following income and expense items for the current year: Wages $ 55,000 Long-term capital loss 4,000 Deductible IRA contribution (Tom is notcovered by a retirement plan at work) 2,000 Mortgage interest on personal residence 6,000 Medical expenses not covered by insurance 4,000 Tom's standard deduction amount 12,550 What is the maximum amount that Tom can contribute to his local church and the full amount be deductible in this year? $31,850 $34,300 $35,750 $50,000

$50,000 For 2020 and 2021, based on the CARES (Coronavirus Aid, Relief, and Economic Security) Act and Consolidated Appropriations Act, 2021 (CAA) provisions, the taxpayer can contribute up to 100% of his adjusted gross income (AGI) in cash charitable contributions. Starting in 2022, these contributions will revert to being limited to 60% of AGI. In this problem, the standard deduction yields more tax benefit than itemizing and the AGI is calculated as wages ($55,000) less long-term capital loss (limited to $3,000) less the contribution to the individual retirement account (IRA) ($2,000), which is $50,000. Note: The CARES Act waived the 60%-of-AGI limit for cash donations made in tax year 2020 only, instead allowing for cash contributions of up to 100% of AGI. This provision was extended to tax year 2021 by the CAA.

Gene and Olive Olson are married, under age 50, and file a joint return in 2021. The Olsons are both active participants in qualified retirement plans. The Olsons have adjusted gross income (AGI) of $115,000 for 2021 and each contributed $6,000 to a traditional IRA. What is the deduction for IRA contributions for the Olsons in 2021? $0 $6,000 $5,500 $12,000

$6,000 In 2021, the phaseout of the IRA deduction for married taxpayers participating in another pension plan filing jointly exists for AGI between $105,000 and $125,000. Since their AGI is halfway between $105,000 and $125,000, the phaseout range, only half of the combined contribution of $12,000 is deductible ($6,000).

Lane, Inc., an S corporation, pays single coverage health insurance premiums of $4,800 per year and family coverage premiums of $7,200 per year for each eligible employee. Mill is a 10% shareholder-employee in Lane. On Mill's behalf, Lane pays Mill's family coverage under the health insurance plan. What amount of insurance premium is includible in Mill's gross income? $0 $720 $4,800 $7,200

$7,200 If Mill owned less than 2% of the company, the correct answer would be zero. Since Mill owns more than 2% of the company, the insurance premiums paid by the S corporation are fully taxable. Mill received family benefits of $7,200, which are taxable. Revenue Ruling 91-26

Evan, an individual, has a 40% interest in EF, an S corporation. At the beginning of the year, Evan's basis in EF was $2,000. During the year, EF distributed $100,000 and reported operating income of $200,000. What amount should Evan include in gross income? $38,000 $40,000 $80,000 $118,000

$80,000 Shareholders of an S corporation include a pro rata share of the corporation's nonseparately and separately stated items of income or expense on their personal tax return. In this case, Evan includes 40% of EF's operating income or $80,000 ($200,000 × 0.40 = $80,000) on his personal tax return. None of Evan's $40,000 share of distributions ($100,000 × 0.40 = $40,000) would be taxable since his distributions did not exceed his new basis. Beginning basis $ 2,000 Share of operating income 80,000 Share of distributions (40,000) Ending basis $42,000 In determining whether a distribution exceeds basis (and is taxable), the ordering rules allow basis to be increased by current-year income before reducing basis by distributions.

What qualifies as an organizational expense?

-Accounting and legal fees -Organizational meetings -Fees paid to the state to obtain a corporate charter NOT -Stock issuance costs (printing, professional fees, commissions, and charges for listing stock on an exchange)

Organizational Expenses - Special Rules

-Amortized over 180 months (15 years) -Immediate expensing of first $5,000* *phased out dollar for dollar if total exceeds $50,000 therefore if total is greater than $55,000 NONE of the organizational expenses can be immediately expensed

Partnership - Partner treatment of liquidation of partnership interest

-Capital gain* only on the amount that cash exceeds the basis of the partnership interest -If gain is recognized, the basis of all other property received is zero *Except to the extent of unrealized receivable or inventory - the gain on these items are ordinary income

Start-up expenses

-Costs related to the creation of a business prior to the time the activity becomes a trade or business

Partnership - Sale/exchange of partnership interest

-Generally treated as the sale or exchange of a capital asset

Partnership - When would income be recognized by partner who contributes property as a way to increase their investment?

-If they receive a capital interest in the partnership in exchange for services rendered -Partner recognizes ordinary income at the FMV of the services rendered

Partnership - What increases a partner's basis

-Investment of property or cash (limited to the partner's basis in the property - carryover basis/holding period) -Taxable income -Tax-exempt income -Excess depletion deductions -Increase in partnership liabilities

Partnership - What decreases a partner's basis

-Withdrawals of money -Adjusted basis of all other property distributed to the partner -Partnership losses (including capital losses) -Nondeductible partnership expenditures -Partner's basis can NEVER go below zero -Assumption by partnership of a partner's personal liability (by the amount of the liability assumed by the other partners)

Leah, a freelance writer, uses the cash method for business. The tax year ends on December 31. Which of the following should be included in the determination of Leah's gross income for the current year? Leah received a client check on January 2, dated December 26. Leah received a check from a client on December 28 which was returned for insufficient funds on January 2. Leah received a dividend check on January 5 of the following year. The dividends were declared payable on December 30 of the current year. A client notified Leah on December 27 of the current year that a check was ready. She did not pick up the check until January 4 of the following year.

A client notified Leah on December 27 of the current year that a check was ready. She did not pick up the check until January 4 of the following year. A cash-basis taxpayer should report income for the year in which it is either actually or constructively received. Income is constructively received by a taxpayer in the taxable year during which it is credited to his account, set apart for him, or otherwise made available so that he may draw upon it at any time. Income is not constructively received if the taxpayer's control of its receipt is subject to substantial limitations or restrictions. The dividends declared on December 30 of the current year by an unrelated corporation would not be deemed constructively received. Regulation Section 1.451-2(a) and (b)

For which of the following contracts will a court generally grant the remedy of specific performance? A contract for the sale of a patent A contract of employment A contract for the sale of fungible goods A contract for the sale of stock that is traded on a national stock exchange

A contract for the sale of a patent Specific performance forces performance of the contract when the goods are unique. Patents are unique, so the court would grant the remedy for a contract for the sale of a patent.

Which of the following rights is considered intangible personal property? An easement A contract right Both an easement and a contract right Neither an easement nor a contract right

A contract right A contract right is an intangible asset and is also personal property, so it qualifies as intangible personal property. An easement is an intangible asset since it is a right but has no physical existence. However, it is a right in real property, so it cannot be personal property.

Which of the following statements is correct regarding the deductibility of an individual's medical expenses? A medical expense paid by credit card is deductible in the year the credit card bill is paid. A medical expense deduction is allowed for payments made in the current year for medical services received in earlier years. Medical expenses, net of insurance reimbursements, are disregarded in the alternative minimum tax calculation. A medical expense deduction is not allowed for Medicare insurance premiums.

A medical expense deduction is allowed for payments made in the current year for medical services received in earlier years. Expenses are considered as paid at the time of the credit card transaction regardless of the timing of the payment of the credit card by the taxpayer. The expenditure and the borrowing of the funds occur simultaneously. These shall be allowed as a deduction of the expenses paid during the taxable year, not compensated for by insurance or otherwise, for medical care. The definition of "medical care" includes amounts paid for insurance premiums under the Social Security Act (Title XVIII, Part B). Medical expenses are deductible under the alternative minimum tax as an itemized deduction for amounts that exceed the appropriate percentage of adjusted gross income (AGI). (The 7.5%-of-AGI limitation was made permanent by the Consolidated Appropriations Act, 2021 (CAA)).

Which of the following types of mistake will generally make a contract unenforceable and allow it to be rescinded? A unilateral mistake of fact A mutual mistake of fact A unilateral mistake of value A mutual mistake of value

A mutual mistake of fact A mistake of fact is one that is important to the subject matter of the contract. If both parties are mistaken as to a material fact in the contract (mutual mistake of fact), then the contract can be rescinded by either party because there has been no agreement. A unilateral mistake of fact (when one contracting party makes a mistake as to a material fact) does not usually afford the mistaken party any right to relief from the contract. A mistake in value or quality, whether mutual or unilateral, cannot generally make a contract unenforceable, so long as one party has not entered the contract to take advantage of the mistaken party.

In the current year, an unmarried individual with modified adjusted gross income of $25,000 paid $1,000 interest on a qualified education loan entered into on July 1. How may the individual treat the interest for income tax purposes? As a $500 deduction to arrive at AGI for the year As a $1,000 deduction to arrive at AGI for the year As a $1,000 itemized deduction As a nondeductible item of personal interest

As a $1,000 deduction to arrive at AGI for the year Individual taxpayers may deduct up to $2,500 of qualified student loan interest as an above-the-line deduction to arrive at AGI. This amount is phased out for higher-income taxpayers. The deduction is not available for taxpayers who are dependents of others or for taxpayers using the married filing separately filing status. The phaseout does not apply to this taxpayer. IRC Section 221

O'Brien purchased two automobiles for personal use. Automobile 1 had an adjusted basis of $20,000, and automobile 2 had an adjusted basis of $10,000. O'Brien sold automobile 1 for $15,000 and automobile 2 for $15,000. What gain or loss should O'Brien recognize on the sales of the automobiles? Automobile 1: loss of $5,000Automobile 2: gain of $5,000 Automobile 1: loss of $0Automobile 2: gain of $5,000 Automobile 1: loss of $5,000Automobile 2: gain of $0 Automobile 1: loss of $0Automobile 2: gain of $0

Automobile 1: loss of $0Automobile 2: gain of $5,000 Generally, gains on property transactions are recognized but losses on the sale, exchange, or condemnation of personal-use assets are not recognized. O'Brien had a $5,000 loss ($20,000 basis − $15,000 sale price) on automobile 1, which is nondeductible, and a gain of $5,000 ($10,000 basis − $15,000 sale price) on automobile 2, which is taxable. The gain and loss cannot be offset.

Which of the following inventory methods is prohibited? Fair market value for securities dealers Selling price, less direct cost of disposition for goods unsalable at normal prices Base-stock Lower of cost or market

Base-stock Under the base-stock method, a constant or nominal price is used under the assumption that a "normal quantity" of inventory is on hand. This is not an acceptable method.

Partnership - Partners ending capital account balance formula

Beginning balance + Capital contributions +/- Current year income +/- Other increases (decreases) - Withdrawals/distributions Ending capital balance

Micro Corp., a calendar-year accrual-basis corporation, purchased a 5-year, 8%, $100,000 taxable corporate bond for $108,530, on July 1, Year 11, the date the bond was issued. The bond paid interest semiannually. Micro elected to amortize the bond premium. For Micro's Year 15 tax return, the bond premium amortization for Year 15 should be: I computed under the constant yield to maturity method. II treated as an offset to the interest income on the bond. I only II only Both I and II Neither I nor II

Both I and II For Micro's Year 15 tax return, the bond premium amortization for Year 15 should be: computed under the constant yield to maturity method and treated as an offset to the interest income on the bond. When a corporation pays a premium (an amount paid in excess of the bond's fair amount) for a taxable bond, it has the option of (1) amortizing the premium until the bond matures and reducing the basis of the bond by the amortized amount (which will offset the interest income on the bond) or (2) not amortizing the premium (this means the premium is part of the bond basis). For bonds issued after September 27, 1985, the premium amortization is calculated under a "constant yield method." IRC Section 171(a) and (b)(3) Since Micro elected to amortize the bond premium, the premium amortization should be computed under the "constant yield to maturity method" and treated as an offset to the interest income on the bond.

Micro Corp., a calendar-year accrual-basis corporation, purchased a 5-year, 8%, $100,000 taxable corporate bond for $108,530, on July 1, Year 11, the date the bond was issued. The bond paid interest semiannually. Micro elected to amortize the bond premium. For Micro's Year 15 tax return, the bond premium amortization for Year 15 should be: I computed under the constant yield to maturity method. II treated as an offset to the interest income on the bond. I only II only Both I and II Neither I nor II

Both I and II For Micro's Year 15 tax return, the bond premium amortization for Year 15 should be: computed under the constant yield to maturity method and treated as an offset to the interest income on the bond. When a corporation pays a premium (an amount paid in excess of the bond's fair amount) for a taxable bond, it has the option of (1) amortizing the premium until the bond matures and reducing the basis of the bond by the amortized amount (which will offset the interest income on the bond) or (2) not amortizing the premium (this means the premium is part of the bond basis). For bonds issued after September 27, 1985, the premium amortization is calculated under a "constant yield method." IRC Section 171(a) and (b)(3) Since Micro elected to amortize the bond premium, the premium amortization should be computed under the "constant yield to maturity method" and treated as an offset to the interest income on the bond.

DAC Foundation awarded Kent $75,000 in recognition of lifelong literary achievement. Kent was not required to render future services as a condition to receive the $75,000. What condition(s) must have been met for the award to be excluded from Kent's gross income? Kent was selected for the award by DAC without any action on Kent's part. Pursuant to Kent's designation, DAC paid the amount of the award either to a governmental unit or to a charitable organization. I only II only Both I and II Neither I nor II

Both I and II The general rule is that prizes and awards are taxable. However, if you meet the following requirements, you can exclude the prize from gross income: The prize must be made primarily in recognition of religious, educational, artistic, charitable, scientific, literary, or civic achievement. The recipient was selected through no action on his part. Substantial future services are not required. The prize or award is transferred by the payor to a governmental unit or charitable organization based on direction from the recipient. All of the requirements must be met in order to exclude the prize. IRC Section 74(b)

Which of the following prejudgment remedies would be available to a creditor when a debtor owns no real property? A writ of attachment Garnishment Both a writ of attachment and garnishment Neither a writ of attachment nor garnishment

Both a writ of attachment and garnishment An unpaid creditor may seek a court order requiring the debtor's assets be seized and held to satisfy an unpaid obligation. This may include both a garnishment of the debtor's wages, as well as a writ of attachment over the debtor's property. The writ of attachment may encompass both real and personal property. In this case, the fact that the debtor owns no real property will not prevent the creditor from obtaining a writ of attachment over his personal property. A writ of attachment will require a court order, and a garnishment may or may not, depending on the jurisdiction.

When calculating the maximum allowable contribution made to a health savings account that is excludible from a taxpayer's income, what should one consider? Both employer and employee contributions are combined to determine the maximum allowable contribution. Only employee contributions are considered when calculating the maximum contribution. Only the employer contributions are excludible from taxable income. There is no amount that is excludible from income.

Both employer and employee contributions are combined to determine the maximum allowable contribution. Both employer and employee contributions to an employee's health savings account are combined to determine the maximum allowable contribution that is excludible from income.

Partnership - Distribution - Cash > Basis of partnership interest

Capital gain

In year 2, Carson was hired as an employee of Barton Co. As part of his employment contract, Barton provided a company car for Carson's spouse, Mary, who is not employed. The value for the use of the automobile in year 2 was $8,000. Carson does not use the automobile. Carson and Mary file separate individual income tax returns. What amounts, if any, should be reported as the value of a taxable fringe benefit on Carson's or Mary's year 2 income tax return for personal use of the automobile? Carson $4,000; Mary $4,000 Carson $0; Mary $0 Carson $0; Mary $8,000 Carson $8,000; Mary $0

Carson $8,000; Mary $0 Effective January 1, 2018, no deduction is allowed for any employer expense incurred for providing any transportation, or any payment or reimbursement for transportation fringe benefits, except as necessary to ensure the safety of the employee. Therefore, since the fringe benefit is a result of Carson's employment (not his wife's), Carson should include the full $8,000 value as a taxable benefit.

Partnership - Partners capital contribution

Cash + Adjusted basis - Liabilities assumed by partnership Amount of capital contribution

Partnership - Withdrawals and distributions

Cash + Adjusted tax basis of all property distributed by the partnership to the partner =Total withdrawal/distribution

Partnership - How to determine gain/loss on sale/exchange of partnership interest

Cash + FMV of any other property received* + Partner's share of partnership liabilities the partner will no longer be responsible for - Adjusted basis including share of partnership liabilities Capital gain/loss recognized *Unrealized receivables and inventory are treated as ordinary income

Which of the following actions between a debtor and its creditors will generally cause the debtor's release from its debts? Composition of creditors Assignment for the benefit of creditors Both composition of creditors and assignment for the benefit of creditors Neither composition of creditors nor assignment for the benefit of creditors

Composition of creditors In a composition agreement, a debtor enters into an agreement with his creditors whereby the debtor agrees to pay the creditors some fraction of the amount of the outstanding debt. Such an agreement discharges the debtor once the debt is actually paid. An assignment for the benefit of creditors is a transfer by the debtor of all of his assets to a trustee, with the assigned funds to be used to pay off outstanding debts. This does not have the effect of releasing the debtor from the debts.

Which of the following rights does one co-surety generally have against another co-surety? Contribution Subrogation Reimbursement Exoneration

Contribution When two or more sureties exist, a surety who has paid more than his or her agreed share is entitled to reimbursement from the co-surety in accordance with the surety contract. This right to receive payment from co-sureties is known as "contribution."

Nan, a cash-basis taxpayer, borrowed money from a bank and signed a 10-year interest-bearing note on business property on January 1 of the current year. The cash flow from Nan's business enabled Nan to prepay the first 3 years of interest attributable to the note on December 31 of the current year. How should Nan treat the prepayment of interest for tax purposes? Deduct the entire amount as a current expense Deduct the current year's interest and amortize the balance over the next 2 years Capitalize the interest and amortize the balance over the 10-year load period Capitalize the interest as part of the basis of the business property

Deduct the current year's interest and amortize the balance over the next 2 years Although Nan is a cash-basis taxpayer, prepaid expenses of over a year are handled differently. Nan paid 3 years of the interest, so she will only expense the first year in the current year and the next 2 years will not be deducted until years 2 and 3. Therefore, the correct answer is to deduct the current year's interest and amortize the balance over the next 2 years. Deducting the entire amount as a current expense is incorrect as the amount covers 3 years and only the current year can be deducted. Capitalizing the interest and amortizing the balance over the 10-year loan period is incorrect as Nan did not pay 10 years of interest. Capitalizing the interest as part of the basis of the business property is incorrect as Nan cannot capitalize as part of the basis of the property.

Tom Lewis, an individual taxpayer, paid an annual personal property tax amount based on the value of his car. Select the appropriate tax treatment on Tom's tax return. Not deductible on Form 1040 Deductible in full on Schedule A—Itemized Deductions Deductible on Schedule A—Itemized Deductions, subject to a threshold amount of 2% of adjusted gross income Deductible on Schedule A—Itemized Deductions, subject to a $500 floor and a threshold of 10% of adjusted gross income

Deductible in full on Schedule A—Itemized Deductions Payment for registration and licensing of a car may be deductible as a personal property tax only if it is imposed annually and assessed in proportion to the value of the car. Deductible taxes (such as personal property tax) that are not directly connected with a trade or business (or with property held for the production of rents and royalties) may be deducted in full as an itemized deduction.

To what extent is the fee paid to a trustee of a trust deductible on Form 1041? Fully deductible Not deductible Deductible to the extent of ratio of taxable income to total income 50% deductible

Deductible to the extent of ratio of taxable income to total income Indirect expenses of a trust such as trustee fees are considered to apply to all income. The ratio of taxable income to total income (not including income allocated to corpus) is used to determine the deduction. For example, if a trust had total income of $20,000, of which $15,000 was taxable and $5,000 was tax-exempt interest, then 75% of the trustee fees would be deductible ($15,000 taxable ÷ $20,000 total = 0.75).

Dye sent Hill a written offer to sell a tract of land located in Newtown for $60,000. The parties were engaged in a separate dispute. The offer stated that it would be irrevocable for 60 days if Hill would refrain from suing Dye during this time. Hill promptly delivered a promise not to sue during the term of the offer and to forego suit if Hill accepted the offer. Dye subsequently decided that the possible suit by Hill was groundless and therefore phoned Hill and revoked the offer 15 days after making it. Hill mailed an acceptance on the 20th day. Dye did not reply. Under the circumstances: Dye's offer was supported by consideration and was not revocable when accepted. Dye's written offer would be irrevocable even without consideration. Dye's silence was an acceptance of Hill's promise. Dye's revocation, not being in writing, was invalid.

Dye's offer was supported by consideration and was not revocable when accepted. One of the elements of a contract is consideration. Hill's promise not to sue was consideration. An irrevocable offer is considered accepted when consideration is received.

Bixler obtained an option on a building he believed was suitable for use by a corporation he and two other individuals were organizing. After the corporation was successfully promoted, Bixler met with the board of directors, who agreed to acquire the property for $200,000. Bixler deeded the building to the corporation and the corporation began business in it. Bixler's option contract called for the payment of only $155,000 for the building and he purchased it for that price. When the directors later learned that Bixler paid only $155,000, they demanded the return of Bixler's $45,000 profit. Bixler refused, claiming the building was worth far more than $200,000 both when he secured the option and when he deeded it to the corporation. Which of the following statements correctly applies to Bixler's conduct? It was improper for Bixler to contract for the option without first having secured the assent of the board of directors. If, as Bixler claimed, the building was fairly worth more than $200,000, Bixler is entitled to retain the entire price. Even if, as Bixler claimed, the building was fairly worth more than $200,000, Bixler nevertheless must return the $45,000 to the corporation. In order for Bixler to be obligated to return any amount to the corporation, the board of directors must establish that the building was worth less than $200,000.

Even if, as Bixler claimed, the building was fairly worth more than $200,000, Bixler nevertheless must return the $45,000 to the corporation. The directors of a corporation have a fiduciary responsibility to the corporation. They have the duty of loyalty—the duty to subordinate personal interests to those of the corporation and its shareholders. Usurping a corporate opportunity is a violation of a director's duty of loyalty. If usurpation is proven, the corporation can recover any profits made by the director or officer.

Which of the following organizations would not qualify for exemption from federal income tax? College alumni association Social clubs that allow only limited usage by general public Fraternal society not operating under a lodge system Political organization

Fraternal society not operating under a lodge system Under IRC Section 501(c)(10), domestic fraternal societies must operate under the lodge system to be exempt from federal income taxation. College alumni associations generally qualify for exemption from federal income tax under IRC Section 501(c)(3). If it does not meet the characteristics required by IRC Section 501(c)(3), it may still be exempt as a social club if it meets the requirements described by IRC Section 501(c)(7). A social or recreation club under IRC Section 501(c)(7) is allowed to receive up to 35% of its gross receipts from sources outside of its membership without losing its status as a tax-exempt organization. The club must have an established membership, which must be limited in some manner. Under IRC Section 527, a political organization is considered a tax-exempt organization. It is subject to tax only on nonexempt income. Examples of exempt income include contributions, membership dues, and proceeds from a political fund-raising event. IRS Publication 557, chapter 4 (Rev 10-2013); IRC Section 501(c)

Under the Uniform Partnership Act, which of the following statements is (are) correct regarding the effect of the assignment of an interest in a general partnership? I. The assignee is personally responsible for the assigning partner's share of past and future partnership debts. II. The assignee is entitled to the assigning partner's interest in partnership profits and surplus on dissolution of the partnership. II only I only Both I and II Neither I nor II

II only Assignment transfers rights under a contract to another (not liabilities). The assignee obtains the right to the assignor's share of partnership profits and what the assignor would receive if the partnership would be dissolved.

Which of the following statements is correct regarding the parol evidence rule? It applies only in cases involving an oral contract. It applies only to subsequent written modifications to a written contract. It applies to subsequent oral agreements that contradict the terms of a final written agreement. It applies to prior or contemporaneous oral agreements that contradict the terms of final written agreements.

It applies to prior or contemporaneous oral agreements that contradict the terms of final written agreements. Parol evidence is extrinsic (oral or written) evidence about the agreement that is not included in the final written agreement (i.e., it is outside the agreement and usually oral). The parol evidence rule excludes the admission of such evidence in a court of law; in other words, once the parties have a written and signed contract, evidence of prior or contemporaneous oral agreements that contradict the terms of the final written agreement (i.e., parol evidence) are not admissible for the purpose of varying or contradicting what is written into the contract.

Which of the following statements is correct regarding the parol evidence rule? It applies only in cases involving an oral contract. It applies only to subsequent written modifications to a written contract. It applies to subsequent oral agreements that contradict the terms of a final written agreement. It applies to prior or contemporaneous oral agreements that contradict the terms of final written agreements.

It applies to prior or contemporaneous oral agreements that contradict the terms of final written agreements. Parol evidence is extrinsic (oral or written) evidence about the agreement that is not included in the final written agreement (i.e., it is outside the agreement and usually oral). The parol evidence rule excludes the admission of such evidence in a court of law; in other words, once the parties have a written and signed contract, evidence of prior or contemporaneous oral agreements that contradict the terms of the final written agreement (i.e., parol evidence) are not admissible for the purpose of varying or contradicting what is written into the contract.

Noll gives Carr a written power of attorney. Which of the following statements is correct regarding this power of attorney? It must be signed by both Noll and Carr. It must be for a definite period of time. It may continue in existence after Noll's death. It may limit Carr's authority to specific transactions.

It may limit Carr's authority to specific transactions. A power of attorney delegates authority from the principal (Noll) to the agent (Carr). This authority may be general or it may be limited or specific. It must be signed only by the principal (the agent need not sign the power of attorney), and it has force and effect for an indefinite time, unless otherwise stated, but will not be effective after the death of the principal. (A Last Will and Testament is required or some other testamentary document such as a trust is required for disposition after death.)

Downs, Frey, and Vick formed the DFV general partnership to act as manufacturers' representatives. The partners agreed Downs would receive 40% of any partnership profits, and Frey and Vick would each receive 30% of such profits. It was also agreed that the partnership would not terminate for five years. After the fourth year, the partners agreed to terminate the partnership. At that time, the partners' capital accounts were as follows: Downs, $20,000; Frey, $15,000; and Vick, $10,000. There also were undistributed losses of $30,000. Which of the following statements about the form of the DFV partnership agreement is correct? It must be in writing because the partnership was to last for longer than one year. It must be in writing because partnership profits would not be equally divided. It could be oral because the partners had explicitly agreed to do business together. It could be oral because the partnership did not deal in real estate.

It must be in writing because the partnership was to last for longer than one year. Generally, a partnership agreement is not required. As long as there is no dispute between the partners, it is not necessary to have an agreement in writing because partnership profits would not be equally divided, in an oral arrangement because the partners had explicitly agreed to do business together, or in an oral agreement because the partnership did not deal in real estate. The planned termination of a partnership is unusual and would necessitate a partnership agreement. For the agreement to be enforceable, a written agreement is required under the statute of frauds.

Jorge's principal place of business is considered to be at his personal residence (i.e., he is self-employed and qualifies to deduct home-office expenses). On occasion, Jorge travels from his residence to various temporary work sites. In determining his deduction for transportation expenses, which of the following statements is correct? Jorge may not include the expenses that were incurred in traveling to work sites inside the metropolitan area in which he lives. Jorge may not include the expenses that were incurred in traveling to work sites outside the metropolitan area in which he lives. Jorge may only include expenses that were incurred in traveling to regular (nontemporary) job sites. Jorge may include the expenses that were incurred in traveling to any work location in the same trade or business.

Jorge may include the expenses that were incurred in traveling to any work location in the same trade or business. If a taxpayer's residence is his or her principal place of business for purposes of IRC Section 280A(c)(1)(A) (i.e., he or she is self-employed and qualifies for the home-office deduction), the taxpayer may deduct daily transportation expenses incurred in going between the residence and another work location in the same trade or business, regardless of whether the other work location is regular or temporary and regardless of the distance.

Brown cosigned Royal's $50,000 note to State Bank. If Royal is later adjudicated mentally incompetent, what would be Brown's liability on the note? Liable to pay State on the due date of the note Liable to pay State only if State first seeks payment from Royal Not liable to pay State because Royal's incompetency discharges Royal as a surety Not liable to pay State unless Brown was a compensated surety

Liable to pay State on the due date of the note Brown is liable for his own actions. If Brown signed a note to State Bank for $50,000, then Brown is liable for the $50,000. Royal's mental competency is moot in regard to Brown. However, Royal's mental competency is in fact relevant in regard to himself. If Royal was mentally incompetent at the time he signed the note, then Royal is not liable for the debt. If you are confused, consider a rephrase of this question: Brown cosigned her 16-year-old daughter's car loan for $50,000. Because the daughter is a minor, and thereby incompetent in the eyes of the law, does Brown still have to pay? Absolutely—which is why the car dealership will be certain to have Brown cosign the note.

Which of the following parties generally has the most management rights? Minority shareholder in a corporation listed on a national stock exchange Limited partner in a general partnership Member of a limited liability company Limited partner in a limited partnership

Member of a limited liability company Out of the choices provided, a member of a limited liability company generally has the most management rights. A limited partner, in order to maintain limited liability, can have no management rights. Shareholders in a corporation elect the board of directors, who then make management decisions. Shareholders have no management rights.

Mike, an employee of a large corporation, typically works at corporate headquarters. Mike is assigned to work temporarily at a site away from corporate headquarters for a period of two months. In determining his deduction for transportation expenses, which of the following statements is correct? Mike may only deduct transportation expenses incurred in traveling from his personal residence to the temporary work site if the work site is outside of his metropolitan area. Mike may only deduct transportation expenses incurred in traveling from his personal residence to the temporary work site if the work site is inside of his metropolitan area. Mike may not deduct transportation expenses incurred in traveling from his personal residence to the temporary work site regardless of the distance. Mike may only deduct transportation expenses incurred in traveling from his personal residence to the temporary work site if his residence qualifies for the home-office deduction.

Mike may not deduct transportation expenses incurred in traveling from his personal residence to the temporary work site regardless of the distance. Under the Tax Cuts and Jobs Act of 2017 (TCJA), itemized miscellaneous deductions taken under the 2% threshold are no longer deductible. Mike may not deduct any of his transportation costs.

Which of the following is (are) among the requirements to enable a taxpayer to select the qualifying widow(er) filing status on IRS Form 1040 U.S. individual income tax return? I A dependent has lived with the taxpayer for six months. II The taxpayer has maintained the cost of the principal residence for six months. I only II only Both I and II Neither I nor II

Neither I nor II IRS Form 1040 uses the term "qualifying widow(er)" in place of "surviving spouse." The term "surviving spouse" means a taxpayer whose spouse died during either of the two taxable years immediately preceding the current taxable year, and who maintains his or her home where a dependent resides. Dependent in this case means a son, stepson, daughter, or stepdaughter of the taxpayer, and the taxpayer is entitled to a dependent deduction for the taxable year. The choices given in this question were not correct because the home and residence of the dependent must be maintained for the full year. Exceptions to the full-year rule are if the dependent is born or dies during the year.

Which of the following activities regularly conducted by a tax-exempt organization will result in unrelated business income? I. Selling articles made by handicapped persons as part of their rehabilitation, when the organization is involved exclusively in their rehabilitation II. Operating a grocery store almost fully staffed by emotionally handicapped persons as part of a therapeutic program I only II only Both I and II Neither I nor II

Neither I nor II Neither of the following activities regularly conducted by a tax-exempt organization will result in unrelated business income: Selling articles made by handicapped persons as part of their rehabilitation, when the organization is involved exclusively in their rehabilitation. Operating a grocery store almost fully staffed by emotionally handicapped persons as part of a therapeutic program. Unrelated business taxable income must be derived from an activity that constitutes a trade or business that is regularly carried on and is not substantially related to the organization's tax-exempt purposes. Generally, a small-scale operation will be exempt but as growth causes the operation to grow to large-scale, the IRS may challenge it as UBIT. Revenue Rulings 76-94, 68-581, and 75-472; IRC Section 512

Which of the following individuals are eligible for the earned income credit? I Tom and Jane Smith, both age 55, are a married couple and are filing a joint return. Their modified adjusted gross income for the year is below the threshold amount to be eligible for the earned income credit. Their only source of income for the year is their retirement pension. II Mike and Ann Jones, both age 50, are a married couple and are filing a joint return. Their modified adjusted gross income for the year is below the threshold amount to be eligible for the earned income credit. Mike and Ann's sources of income for the year are wages for both and $10,100 of tax-exempt interest. I only II only Both I and II Neither I nor II

Neither I nor II Tom and Jane are not eligible for the earned income credit since their only source of income is their retirement pension. A pension is not considered to be "earned income." An individual must have at least some "earned income" in order to be eligible for the earned income credit. Mike and Ann are not eligible for the earned income credit. Tax-exempt interest is considered "disqualified income" under IRC Section 32(a). Disqualified income in post-1995 tax years includes an individual's capital gain net income and net passive income in addition to interest, dividends, tax-exempt interest, and nonbusiness rents or royalties. For 2021, the American Rescue Plan Act (ARPA) increased the investment income a taxpayer may have before being disqualified from claiming the earned income credit to $10,000, but the couple's investment income exceeds this amount as well ($10,100). Therefore, they do not qualify to claim the credit.

Partnership - Treatment of partner increases in investment through contributing property

No gain or loss is recognized by the partner or partnership* *Income may be recognized by a partner who receives a capital interest in the partnership in exchange for services rendered; FMV of services rendered would be considered compensation (ordinary income)

Sand orally promised Frost a $10,000 bonus, in addition to a monthly salary, if Frost would work two years for Sand. If Frost works for the two years, will the statute of frauds prevent Frost from collecting the bonus? No, because Frost fully performed No, because the contract did not involve an interest in real estate Yes, because the contract could not be performed within one year Yes, because the monthly salary was the consideration of the contract

No, because Frost fully performed The statute of frauds applies only to contracts for the sale of goods, a transfer of interest in land, promises to pay for the debts of another, or contracts that cannot be performed within one year. Consequently, the statute of frauds will not be considered in the performance of this contract. Frost did fully perform and therefore a contract has been created. A contract contains an offer, acceptance of the offer, and valid consideration. Sand offered Frost a bonus, Frost agrees to work the two years (acceptance), and the $10,000 is the motivation or consideration for the contract.

Drew bought a computer for personal use from Hale Corp. for $3,000. Drew paid $2,000 in cash and signed a security agreement for the balance. Hale properly filed the security agreement. Drew defaulted in paying the balance of the purchase price. Hale asked Drew to pay the balance. When Drew refused, Hale peacefully repossessed the computer. Under the UCC Secured Transactions Article, which of the following remedies will Hale have? Obtain a deficiency judgment against Drew for the amount owed. Sell the computer and retain any surplus over the amount owed. Retain the computer over Drew's objection. Sell the computer without notifying Drew.

Obtain a deficiency judgment against Drew for the amount owed. The Uniform Commercial Code (UCC) permits a creditor to repossess collateral in the event of the debtor's default. The repossession can be achieved through legal processes or by "self help" (the creditor accomplishes repossession himself). If self help is used, the creditor cannot "breach the peace." Assuming that proper procedures are followed, a creditor may later obtain a deficiency judgment against the debtor if the sale of the collateral did not generate sufficient funds to pay the indebtedness in full. Depending on state law, Hale may have to give Drew a specified number of days to redeem the property. In the unlikely event that the personal property has increased in value, any surplus received above the debt to Hale should be returned to Drew.

For head of household filing status, which of the following costs are considered in determining whether the taxpayer has contributed more than one-half the cost of maintaining the household? A Food consumed in the home B Value of services rendered in the home by the taxpayer Both A and B Neither A nor B Only A Only B

Only A In determining the cost for maintaining a household, only paid costs should be considered. Those costs can include mortgage or rent expense, utility charges, food consumed in the home, and repair and maintenance costs. Any unpaid costs (such as the value of services) are not considered when determining the cost of maintaining the home.

A sole proprietor of a farm implement store sold a truck for $15,000 that had been used to make service calls. The truck cost $30,000 three years ago, and $21,360 depreciation was taken. What is the appropriate classification of the $6,360 gain for tax purposes? Ordinary gain Section 1231 (property used in the trade or business and involuntary conversions) gain Long-term capital gain Short-term capital gain

Ordinary gain IRC Section 1231 property is defined as an asset used in a trade or business subject to depreciation and capital gain treatment would be available. But IRC Section 1231 is modified by IRC Section 1245, which states that personal (versus real) property's depreciation taken must be recaptured as ordinary income first. If a gain still remains after the depreciation recapture, then capital gain treatment is applied. In this example, total depreciation taken of $21,360 is greater than the total gain of $6,360. Therefore, all of the $6,360 gain is ordinary.

A sole proprietor of a farm implement store sold a truck for $15,000 that had been used to make service calls. The truck cost $30,000 three years ago, and $21,360 depreciation was taken. What is the appropriate classification of the $6,360 gain for tax purposes? Ordinary gain Section 1231 (property used in the trade or business and involuntary conversions) gain Long-term capital gain Short-term capital gain

Ordinary gain RC Section 1231 property is defined as an asset used in a trade or business subject to depreciation and capital gain treatment would be available. But IRC Section 1231 is modified by IRC Section 1245, which states that personal (versus real) property's depreciation taken must be recaptured as ordinary income first. If a gain still remains after the depreciation recapture, then capital gain treatment is applied. In this example, total depreciation taken of $21,360 is greater than the total gain of $6,360. Therefore, all of the $6,360 gain is ordinary.

Wages paid for domestic services are subject to special rules for determining whether they are subject to payroll taxes. When are domestic wages subject to federal unemployment tax? Over $1,500 to one employee in a year Over $2,000 total wages in a year Over $1,000 total wages in a quarter Only if requested by employee

Over $1,000 total wages in a quarter Wages paid for domestic services are subject to federal unemployment tax if they exceed $1,000 per quarter, aggregating wages paid to all employees.

On June 15, Peters orally offered to sell a used lawn mower to Mason for $125. Peters specified that Mason had until June 20 to accept the offer. On June 16, Peters received an offer to purchase the lawn mower for $150 from Bronson, Mason's neighbor. Peters accepted Bronson's offer. On June 17, Mason saw Bronson using the lawn mower and was told the mower had been sold to Bronson. Mason immediately wrote to Peters to accept the June 15 offer. Which of the following statements is correct? Mason's acceptance would be effective when received by Peters. Mason's acceptance would be effective when mailed. Peters's offer had been revoked and Mason's acceptance was ineffective. Peters was obligated to keep the June 15 offer open until June 20.

Peters's offer had been revoked and Mason's acceptance was ineffective. To create a contract, the offer must be accepted before a termination of the contract. A sale of the property to another entity would be a termination. An indirect revocation by the offerer, such as Mason being told by Bronson that Bronson bought the mower, is a valid termination.

Noninventory goods were purchased and delivered on June 15, 20X1. Several security interests exist in these goods. Which of the following security interests has priority over the others? Security interest in future goods attached June 10, 20X1 Security interest attached June 15, 20X1 Security interest perfected June 20, 20X1 Purchase money security interest perfected June 24, 20X1

Purchase money security interest perfected June 24, 20X1 Under Section 9-312 of the Uniform Commercial Code (UCC), a purchase money security interest (PMSI) in noninventory (e.g., equipment) takes priority over all other competing security interests in the same collateral if the PMSI is perfected by filing a financing statement properly within a 20-day grace period, that is, within 20 days after June 15, 20X1. Since the PMSI was perfected June 24, it has priority over the non-PMSI security interest filed on June 20. A nonpurchase money security interest is subject to the date order; that is, first to file or perfect wins. The other existing security interests will have priority over any subsequent or inferior security interests.

Which of the following actions may be taken by a corporation's board of directors without stockholder approval? Purchasing substantially all of the assets of another corporation Selling substantially all of the corporation's assets Dissolving the corporation Amending the articles of incorporation

Purchasing substantially all of the assets of another corporation Shareholders are entitled to approve fundamental corporate changes to the entity they own. Selling substantially all of the corporation's assets, dissolving the corporation, and amending the articles of incorporation represent an example of a fundamental corporate change that must be approved by (usually) a 2/3rds majority of the voting shareholders. While normally the board of directors is authorized to obtain the assets of another corporation, it would be good form to have stockholder ratification of this acquisition as well. Usually due to the necessity for speed in the acquisition of the assets, such approval may not be available on a timely basis, assuming that it had been required.

On May 1, Year 5, two months after becoming insolvent, Quick Corp., an appliance wholesaler, filed a voluntary petition for bankruptcy under the provisions of Chapter 7 of the Federal Bankruptcy Code. On October 15, Year 4, Quick's board of directors had authorized and paid Erly $50,000 to repay Erly's April 1, Year 4, loan to the corporation. Erly is a sibling of Quick's president. On March 15, Year 5, Quick paid Kray $100,000 for inventory delivered that day. Which of the following is not relevant in determining whether the repayment of Erly's loan is a voidable preferential transfer? Erly is an insider. Quick's payment to Erly was made on account of an antecedent debt. Quick's solvency when the loan was made by Erly Quick's payment to Erly was made within one year of the filing of the bankruptcy petition.

Quick's solvency when the loan was made by Erly Preferential transfers may be voided if made within 90 days before the bankruptcy filing and if the debtor is insolvent. The time period is 12 months if the transfer was to an insider. Since the loan was made 13 months prior to the bankruptcy, Quick's solvency at the time is not relevant. Because the payment was less than 12 months before the filing, the fact that Erly is an insider is relevant. Preferential transfers include antecedent debt, so the payment is relevant. The payment being made within one year of the filing is relevant because Erly is an insider.

Ingot Corp. lent Flange $50,000. At Ingot's request, Flange entered into an agreement with Quill and West for them to act as compensated co-sureties on the loan in the amount of $100,000 each. Ingot released West without Quill's or Flange's consent, and Flange later defaulted on the loan. Which of the following statements is correct? Flange will be released for 50% of the loan balance. Quill will be liable for 50% of the loan balance. Quill will be liable for the entire loan balance. Ingot's release of West will have no effect on Flange's and Quill's liability to Ingot.

Quill will be liable for 50% of the loan balance. The release of one co-surety without the knowledge and consent of the debtor and other co-surety (without a reservation of rights) generally releases the other co-surety to the extent of the proportionate responsibility of the co-surety who was released. In this case, since each surety is equally responsible for the debt, the release of one will have the effect of releasing the other for 50% of the outstanding debt. (Normally, if the co-surety merely could or would not pay, the other co-surety would be responsible to the creditor for the entire obligation and assume the right of subrogation against both the debtor and the defaulting co-surety.)

Brenda, employed full time, makes beaded jewelry as a hobby. In Year 2, Brenda's hobby generated $2,000 of sales, and she incurred $3,000 of travel expenses. What is the proper reporting of the income and expenses related to the activity? Sales of $2,000 are reported in gross income. Sales of $2,000 are reported in gross income, and $3,000 of expenses is reported as a miscellaneous itemized deduction subject to the 2% limitation. Sales and expenses are netted, and the net loss of $1,000 is reported as a miscellaneous itemized deduction. Sales and expenses are netted and deducted for AGI.

Sales of $2,000 are reported in gross income. Hobbies are activities that are engaged in primarily for personal enjoyment rather than to produce profit. The Tax Cuts and Jobs Act of 2017 (TCJA) completely eliminates the itemized deduction for hobby expenses (along with all other miscellaneous itemized deductions); therefore, Brenda must report all $2,000 as gross income.

Partnership - Partnership basis of contributed property (non personal assets)

Same in the hands of the partnership as it was in the hands of the partner

A CPA sued a former client for nonpayment of the final bill. Although happy with the CPA's performance of services, the client claimed that CPA is not entitled to the final bill payment because the contract between the client and the CPA failed to meet the statute of frauds. The client argues that the contract allowed up to 15 months for the CPA to complete the work, the contract price was well over $5,000, and though the client sent signed checks to the CPA, the client did not sign the contract. Which of the following statements about this situation is correct? The statute of frauds does not apply, allowing enforcement of the contract terms. The statute of frauds does not apply, preventing enforcement of the contract terms. The statute of frauds does apply, and the requirements are not satisfied, thereby preventing enforcement of the contract terms. The statute of frauds does apply, but the requirements are satisfied by the client's signing of the checks, allowing enforcement of the contract terms.

The statute of frauds does not apply, allowing enforcement of the contract terms. The statute of frauds does not apply because the signed checks for services satisfies the written requirement necessary to enforce the contract. This is a written enforceable contract not subject to provisions within the statute of frauds. The CPA can enforce the contract and demand payment.

Which of the following statements is correct with respect to ownership, possession, or access to a CPA firm's audit working papers? Working papers may never be obtained by third parties unless the client consents. Working papers are not transferable to a purchaser of a CPA practice unless the client consents. Working papers are subject to the privileged communication rule which, in most jurisdictions, prevents any third-party access to the working papers. Working papers are the client's exclusive property.

Working papers are not transferable to a purchaser of a CPA practice unless the client consents. Even though CPAs are independent contractors and have legal title to their working papers, their possession is custodial. The working papers cannot be transferred without first obtaining the consent of their client. While working papers are the property of the CPA, not the client, they are not considered "privileged" and a CPA may be required to turn over such documents for peer review or when subject to a subpoena (court order). CPAs do not have legal protection regarding confidentiality as do attorneys. The AICPA Code of Professional Conduct's confidential relationship rule, not the privileged communication rule, prevents third-party access without the client's permission.

Charitable contributions subject to the 60% limit that are not fully deductible in the year made may be: neither carried back nor forward. carried back 3 years or carried forward 15 years. carried forward 5 years. carried forward indefinitely until fully used.

carried forward 5 years. Charitable contributions subject to the 60% limit that are not fully deductible in the year made may be carried forward five years. The amount of the excess that may be deducted in any carryover year is limited to the lesser of (1) the remaining portion of any excess contribution not already deducted, or (2) the amount equal to 60% (or 30% for capital gain carryover) of the taxpayer's AGI after first deducting the sum of the charitable contributions (to which the 60% or 30% limitation applies) paid in the carryover year and any excess contributions that have precedence in order of time over the present carryover. In other words, charitable contribution carryovers follow FIFO (first in, first out) rules.

Dale received $1,000 in the current year for jury duty. In exchange for regular compensation from her employer during the period of jury service, Dale was required to remit the entire $1,000 to her employer. In Dale's income tax return, the $1,000 jury duty fee should be: claimed in full as an itemized deduction. claimed as an itemized deduction to the extent exceeding 2% of adjusted gross income. deducted from gross income in arriving at adjusted gross income. included in taxable income without a corresponding offset against other income.

deducted from gross income in arriving at adjusted gross income. Generally, jury duty pay must be included as other income on IRS Form 1040. However, the taxpayer can deduct the amount of jury duty pay remitted to an employer in exchange for continued regular compensation. The deduction from gross income can be entered on line 24 of Schedule 1 attached to IRS Form 1040 or 1040-SR.

Davis, a sole proprietor with no employees, has a Keogh profit-sharing plan to which he may contribute 20% of his annual earned income. For this purpose, "earned income" is defined as net self-employment earnings reduced by the: deductible Keogh contribution. self-employment tax. self-employment tax and a portion of the deductible Keogh contribution. deductible Keogh contribution and a portion of the self-employment tax.

deductible Keogh contribution and a portion of the self-employment tax. A Keogh plan is a retirement plan for self-employed taxpayers named after U.S. Representative Eugene James Keogh. The term "Keogh" is rarely used now, as the law no longer distinguishes between corporate and other plan sponsors. The maximum contribution is 20% of the taxpayer's annual net self-employment income reduced by the deductible Keogh contribution and 50% of self-employment tax

Under the Secured Transactions Article of the UCC, in a situation where the financing statement contains no special provisions and the debtor has defaulted, the secured party generally has the right to: require the debtor to deliver the collateral to the secured party. peacefully take possession of the collateral without judicial process. sell the collateral without notifying the debtor. retain the collateral and proceed against the debtor for any balance due.

peacefully take possession of the collateral without judicial process. Under the Uniform Commercial Code (UCC), a secured party that has properly filed a financing statement without special provisions has the right to take possession of the collateral through judicial proceedings or independently if it can be done without breaching the peace. The secured party cannot force the debtor to deliver the collateral. Retention of the collateral negates the ability to proceed for the balance due. To proceed for the balance due, the secured party must sell the collateral and notify the debtor. Before selling the property, the secured party most notify the debtor of its decision to retain or sell the collateral.

Dan owes debts to four different creditors. To satisfy these debts, Dan transfers his property to a trustee. The trustee converts the property to cash and pays it to all of the creditors on a pro rata basis. If all of the creditors only receive partial payments of the amounts they are due: the debtor will be released from liability for the balance due unless the creditors expressly reserve the right to collect the full amount. the debtor will be released from liability for the balance due since this is a composition agreement. the debtor will not be released from liability for the balance due because an assignment for the benefit of creditors does not require the consent of the creditors. the debtor will not be released from liability for the balance due unless the trustee agrees to the release.

the debtor will not be released from liability for the balance due because an assignment for the benefit of creditors does not require the consent of the creditors. In an assignment for the benefit of creditors, the debtor transfers property to a trustee who sells the property and uses the funds to make payments to the creditors. Since this arrangement does not involve the consent of the creditors, the debtors will not be released from liability for any balance. Any agreement under which partial payments will release the debtor from further liability must be made between the debtor and the creditors. The trustee has no such power. Finally, a composition agreement involves a direct agreement between the debtor and its creditors and does not generally involve transfer of the debtor's property to a trustee. However, in the above instance, a composition agreement for a lesser payment of individuals claims would, in fact, discharge the balance if it were so agreed.

Kay, an art collector, promised Hammer, an art student, that if Hammer could obtain certain rare artifacts within two weeks, Kay would pay for Hammer's post-graduate education. At considerable effort and expense, Hammer obtained the specified artifacts within the 2-week period. When Hammer requested payment, Kay refused. Kay claimed that there was no consideration for the promise. Hammer would prevail against Kay based on: public policy. quasi contract. unilateral contract. unjust enrichment.

unilateral contract. Hammer would prevail against Kay based on the theory of a unilateral contract. This question tests your knowledge of four contract terms: 1. Unilateral Contract: A contract that is formed when, by the terms of the offer, acceptance is given by performance. That is exactly what happened in this problem. An offer was made to pay for post-graduate education if the prospective student did an act (obtained artifacts within two weeks). When performance was rendered, the contract was accepted, and therefore, is enforceable. 2. Unjust Enrichment: Refers to a theory that permits a court to redress a situation where one party has performed some act for the benefit of another and should receive compensation in the name of justice. 3. Public Policy: Refers to a justification that courts will sometimes use when there is no specific law or prior case dealing with the same set of facts, but when court action can be justified on the basis of common sense interpretation of what the general consensus might be over what a court should do. 4. Quasi Contract: A theory under which courts will apply a contractual remedy when there was actually no formal offer or acceptance (and, therefore, no legal contract).

If a buyer accepts an offer containing an immaterial unilateral mistake, the resulting contract will be: void as a matter of law. void at the election of the buyer. valid as to both parties. voidable at the election of the seller.

valid as to both parties. A material mistake would make the contract invalid. Unless the other party was aware of the mistake or was trying to take advantage of the buyer, a unilateral mistake will not prevent the making of the contract.


संबंधित स्टडी सेट्स

Wellness & Decision Making Health Review

View Set

Unit 29: Three - Phase Transformers

View Set

Common Classroom Questions & Phrases (Español I)

View Set